Dnb July 2017 Mock Test
Quiz-summary
0 of 300 questions completed
Questions:
- 1
- 2
- 3
- 4
- 5
- 6
- 7
- 8
- 9
- 10
- 11
- 12
- 13
- 14
- 15
- 16
- 17
- 18
- 19
- 20
- 21
- 22
- 23
- 24
- 25
- 26
- 27
- 28
- 29
- 30
- 31
- 32
- 33
- 34
- 35
- 36
- 37
- 38
- 39
- 40
- 41
- 42
- 43
- 44
- 45
- 46
- 47
- 48
- 49
- 50
- 51
- 52
- 53
- 54
- 55
- 56
- 57
- 58
- 59
- 60
- 61
- 62
- 63
- 64
- 65
- 66
- 67
- 68
- 69
- 70
- 71
- 72
- 73
- 74
- 75
- 76
- 77
- 78
- 79
- 80
- 81
- 82
- 83
- 84
- 85
- 86
- 87
- 88
- 89
- 90
- 91
- 92
- 93
- 94
- 95
- 96
- 97
- 98
- 99
- 100
- 101
- 102
- 103
- 104
- 105
- 106
- 107
- 108
- 109
- 110
- 111
- 112
- 113
- 114
- 115
- 116
- 117
- 118
- 119
- 120
- 121
- 122
- 123
- 124
- 125
- 126
- 127
- 128
- 129
- 130
- 131
- 132
- 133
- 134
- 135
- 136
- 137
- 138
- 139
- 140
- 141
- 142
- 143
- 144
- 145
- 146
- 147
- 148
- 149
- 150
- 151
- 152
- 153
- 154
- 155
- 156
- 157
- 158
- 159
- 160
- 161
- 162
- 163
- 164
- 165
- 166
- 167
- 168
- 169
- 170
- 171
- 172
- 173
- 174
- 175
- 176
- 177
- 178
- 179
- 180
- 181
- 182
- 183
- 184
- 185
- 186
- 187
- 188
- 189
- 190
- 191
- 192
- 193
- 194
- 195
- 196
- 197
- 198
- 199
- 200
- 201
- 202
- 203
- 204
- 205
- 206
- 207
- 208
- 209
- 210
- 211
- 212
- 213
- 214
- 215
- 216
- 217
- 218
- 219
- 220
- 221
- 222
- 223
- 224
- 225
- 226
- 227
- 228
- 229
- 230
- 231
- 232
- 233
- 234
- 235
- 236
- 237
- 238
- 239
- 240
- 241
- 242
- 243
- 244
- 245
- 246
- 247
- 248
- 249
- 250
- 251
- 252
- 253
- 254
- 255
- 256
- 257
- 258
- 259
- 260
- 261
- 262
- 263
- 264
- 265
- 266
- 267
- 268
- 269
- 270
- 271
- 272
- 273
- 274
- 275
- 276
- 277
- 278
- 279
- 280
- 281
- 282
- 283
- 284
- 285
- 286
- 287
- 288
- 289
- 290
- 291
- 292
- 293
- 294
- 295
- 296
- 297
- 298
- 299
- 300
Information
DNB MOCK TEST
You have already completed the quiz before. Hence you can not start it again.
Quiz is loading...
You must sign in or sign up to start the quiz.
You have to finish following quiz, to start this quiz:
Results
0 of 300 questions answered correctly
Your time:
Time has elapsed
You have reached 0 of 0 points, (0)
Average score |
|
Your score |
|
Categories
- Anaesthesia 0%
- Anatomy 0%
- Biochemistry 0%
- Ent 0%
- Forensic medicine 0%
- Medicine 0%
- Microbiology 0%
- Obgy 0%
- Opthalmology 0%
- Orthopaedics 0%
- Paediatrics 0%
- Pathology 0%
- Pathology 0%
- Pharmacology 0%
- Physiology 0%
- PSM 0%
- Psychiatry 0%
- Radiology 0%
- Surgery 0%
- Surgery 0%
Pos. | Name | Entered on | Points | Result |
---|---|---|---|---|
Table is loading | ||||
No data available | ||||
- 1
- 2
- 3
- 4
- 5
- 6
- 7
- 8
- 9
- 10
- 11
- 12
- 13
- 14
- 15
- 16
- 17
- 18
- 19
- 20
- 21
- 22
- 23
- 24
- 25
- 26
- 27
- 28
- 29
- 30
- 31
- 32
- 33
- 34
- 35
- 36
- 37
- 38
- 39
- 40
- 41
- 42
- 43
- 44
- 45
- 46
- 47
- 48
- 49
- 50
- 51
- 52
- 53
- 54
- 55
- 56
- 57
- 58
- 59
- 60
- 61
- 62
- 63
- 64
- 65
- 66
- 67
- 68
- 69
- 70
- 71
- 72
- 73
- 74
- 75
- 76
- 77
- 78
- 79
- 80
- 81
- 82
- 83
- 84
- 85
- 86
- 87
- 88
- 89
- 90
- 91
- 92
- 93
- 94
- 95
- 96
- 97
- 98
- 99
- 100
- 101
- 102
- 103
- 104
- 105
- 106
- 107
- 108
- 109
- 110
- 111
- 112
- 113
- 114
- 115
- 116
- 117
- 118
- 119
- 120
- 121
- 122
- 123
- 124
- 125
- 126
- 127
- 128
- 129
- 130
- 131
- 132
- 133
- 134
- 135
- 136
- 137
- 138
- 139
- 140
- 141
- 142
- 143
- 144
- 145
- 146
- 147
- 148
- 149
- 150
- 151
- 152
- 153
- 154
- 155
- 156
- 157
- 158
- 159
- 160
- 161
- 162
- 163
- 164
- 165
- 166
- 167
- 168
- 169
- 170
- 171
- 172
- 173
- 174
- 175
- 176
- 177
- 178
- 179
- 180
- 181
- 182
- 183
- 184
- 185
- 186
- 187
- 188
- 189
- 190
- 191
- 192
- 193
- 194
- 195
- 196
- 197
- 198
- 199
- 200
- 201
- 202
- 203
- 204
- 205
- 206
- 207
- 208
- 209
- 210
- 211
- 212
- 213
- 214
- 215
- 216
- 217
- 218
- 219
- 220
- 221
- 222
- 223
- 224
- 225
- 226
- 227
- 228
- 229
- 230
- 231
- 232
- 233
- 234
- 235
- 236
- 237
- 238
- 239
- 240
- 241
- 242
- 243
- 244
- 245
- 246
- 247
- 248
- 249
- 250
- 251
- 252
- 253
- 254
- 255
- 256
- 257
- 258
- 259
- 260
- 261
- 262
- 263
- 264
- 265
- 266
- 267
- 268
- 269
- 270
- 271
- 272
- 273
- 274
- 275
- 276
- 277
- 278
- 279
- 280
- 281
- 282
- 283
- 284
- 285
- 286
- 287
- 288
- 289
- 290
- 291
- 292
- 293
- 294
- 295
- 296
- 297
- 298
- 299
- 300
- Answered
- Review
-
Question 1 of 300
1. Question
The branch arising from the main trunk of mandibular nerve is?
Correct
Incorrect
-
Question 2 of 300
2. Question
Regarding ansa cervicalis, all are true except?
Correct
Incorrect
-
Question 3 of 300
3. Question
Roof the fourth ventricle is formed by all of the following except?
Correct
Incorrect
-
Question 4 of 300
4. Question
Anterior wall of the third ventricle is formed by?
Correct
Incorrect
-
Question 5 of 300
5. Question
Which of the following is true regarding the lateral geniculate body?
Correct
Incorrect
-
Question 6 of 300
6. Question
All are true regarding femoral artery except?
Correct
Incorrect
-
Question 7 of 300
7. Question
Differentiating an ASD from a VSD using a chest X-ray is by detection of?
Correct
Incorrect
-
Question 8 of 300
8. Question
Earliest sign of pathological gastroesophageal reflux in infants is?
Correct
Incorrect
-
Question 9 of 300
9. Question
Most common malignant orbital tumor in children is?
Correct
Incorrect
-
Question 10 of 300
10. Question
Which of the following is the treatment of choice for true precocious puberty?
Correct
Incorrect
-
Question 11 of 300
11. Question
Which of the following is the most common complication of mumps in children?
Correct
Incorrect
-
Question 12 of 300
12. Question
An eight year old boy presented to the casualty with high fever, pruritic erythematous rash, joint pain and lymph node enlargement. There is a history of upper respiratory tract infection for which he was on cefaclor â 8 days completed of a 10 day course. The most likely diagnosis is?
Correct
Incorrect
-
Question 13 of 300
13. Question
All of the following therapies may be required in a 1 hour old infant with severe birth asphyxia except:
Correct
Incorrect
-
Question 14 of 300
14. Question
The following bacteria are most often associated with acute neonatal meningitis except:
Correct
Incorrect
-
Question 15 of 300
15. Question
The most common presentation of a child with Wilms tumor is:
Correct
Incorrect
-
Question 16 of 300
16. Question
A child with recurrent urinary tract infections is most likely to show:
Correct
Incorrect
-
Question 17 of 300
17. Question
Blalock and Taussig shunt is done between:
Correct
Incorrect
-
Question 18 of 300
18. Question
The earliest indicator of response after starting iron in a 6-year-old girl with iron deficiency is:
Correct
Incorrect
-
Question 19 of 300
19. Question
The requirement of potassium is a child is:
Correct
Incorrect
-
Question 20 of 300
20. Question
A 15-year old female presented to the emergency department with history of recurrent epistaxis, hematuria and hematochezia. There was a history of profuse bleeding from the umbilicus stump at birth. Previous investigations revealed normal prothrombin time, activated partial thromboplastin time, thrombin time and fibrinogen levels. Her platelet counts as well as platelet function tests were normal but urea clot lysis test was positive. Which one of the following clotting factor is most likely to be deficient?
Correct
Incorrect
-
Question 21 of 300
21. Question
Which organ is the primary site of hematopoiesis in the fetus before midpregnancy?
Correct
Incorrect
-
Question 22 of 300
22. Question
A baby presenting with multiple deformities, cleft lip, cleft palate, microcephaly, small eyes, scalp defect and polydactyly. Probable diagnosis is?
Correct
Incorrect
-
Question 23 of 300
23. Question
All of the following are risk factors for the development of Legionella pneumonia EXCEPT
Correct
Incorrect
-
Question 24 of 300
24. Question
What is the most common manifestation of Coccidioides infection in an immunocompetent host?
Correct
Incorrect
-
Question 25 of 300
25. Question
A 32-year-old man presents with jaundice and malaise. He is found to have acute hepatitis B with positive hepatitis B virus (HBV) DNA and E antigen. Which of the following antiviral agents are approved as part of a therapeutic regimen for monoinfection with hepatitis B?
Correct
Incorrect
-
Question 26 of 300
26. Question
The evaluation in a newly diagnosed case of acute lymphoid leukemia (ALL) should routinely include all of the following EXCEPT
Correct
Incorrect
-
Question 27 of 300
27. Question
Aplastic anemia has been associated with all of the following EXCEPT
Correct
Incorrect
-
Question 28 of 300
28. Question
All of the following laboratory values are consistent with an intravascular hemolytic anemia EXCEPT
Correct
Incorrect
-
Question 29 of 300
29. Question
All the following match the anticoagulant with its correct mechanism of action EXCEPT
Correct
Incorrect
-
Question 30 of 300
30. Question
All the following are late complications of bone marrow transplant preparative regimens EXCEPT
Correct
Incorrect
-
Question 31 of 300
31. Question
A 29-year-old woman is in the intensive care unit with rhabdomyolysis due to compartment syndrome of the lower extremities after a car accident. Her clinical course has been complicated by acute renal failure and severe pain. She has undergone fasciotomies and is admitted to the intensive care unit. What is the most appropriate course of action at this point?
Correct
Incorrect
-
Question 32 of 300
32. Question
A 54-year-old male with type 2 diabetes mellitus reports 3 months of exertional chest pain. His physical examination is notable for obesity with a body mass index (BMI) of 32 kg/m2, blood pressure of 150/90, an S4, no cardiac murmurs, and no peripheral edema. Fasting glucose is 130 mg/dL, and serum triglycerides are 200 mg/dL. Which of the following is most likely in this patient?
Correct
Incorrect
-
Question 33 of 300
33. Question
A 30-year-old female is seen in the clinic before undergoing an esophageal dilation for a stricture. Her past medical history is notable for mitral valve prolapse with mild regurgitation. She takes no medications and is allergic to penicillin. Her physician should recommend which of the following?
Correct
Incorrect
-
Question 34 of 300
34. Question
Which of the following conditions is not associated with sinus bradycardia?
Correct
Incorrect
-
Question 35 of 300
35. Question
Acute hyperkalemia is associated with which of the following electrocardiographic changes?
Correct
Incorrect
-
Question 36 of 300
36. Question
All of the following clinical findings are consistent with severe mitral stenosis EXCEPT
Correct
Incorrect
-
Question 37 of 300
37. Question
A patient is found to have a holosystolic murmur on physical examination. With deep inspiration, the intensity of the murmur increases. This is consistent with which of the following?
Correct
Incorrect
-
Question 38 of 300
38. Question
A 37-year-old male with Wolff-Parkinson-White syndrome develops a broad-complex irregular tachycardia at a rate of 200 beats per minute. He appears comfortable and has little hemodynamic impairment. Useful treatment at this point might include
Correct
Incorrect
-
Question 39 of 300
39. Question
All the following ECG findings are suggestive of left ventricular hypertrophy EXCEPT
Correct
Incorrect
-
Question 40 of 300
40. Question
A patient is evaluated in the emergency department for peripheral cyanosis. Which of the following is not a potential etiology?
Correct
Incorrect
-
Question 41 of 300
41. Question
A 26-year-old man presents to the clinic with 3 days of severe sore throat and fever. All of the following support the diagnosis of streptococcal pharyngitis EXCEPT
Correct
Incorrect
-
Question 42 of 300
42. Question
A 23-year-old male is climbing a tall mountain. He has no medical problems and takes no medications. Shortly after beginning the climb, he develops severe shortness of breath. Physical examination shows diffuse bilateral inspiratory crackles. Which of the following is the most likely etiology?
Correct
Incorrect
-
Question 43 of 300
43. Question
Which of the following organisms is unlikely to be found in the sputum of a patient with cystic fibrosis?
Correct
Incorrect
-
Question 44 of 300
44. Question
A 35-year-old male is seen in the clinic for evaluation of infertility. He has never fathered any children, and after 2 years of unprotected intercourse his wife has not achieved pregnancy. Sperm analysis shows a normal number of sperm, but they are immotile. Past medical history is notable for recurrent sinopulmonary infections, and the patient recently was told that he has bronchiectasis. Chest radiography is likely to show which of the following?
Correct
Incorrect
-
Question 45 of 300
45. Question
A young man with multiple sclerosis exhibits paradoxical dilation of the right pupil when a flashlight is redirected from the left eye into the right eye. Swinging the flashlight back to the left eye produces constriction of the right pupil. This patient apparently has
Correct
Incorrect
-
Question 46 of 300
46. Question
A 55-year-old man who is extremely obese reports weakness, sweating, tachycardia, confusion, and headache whenever he fasts for more than a few hours. He has prompt relief of symptoms when he eats. These symptoms are most suggestive of which of the following disorders?
Correct
Incorrect
-
Question 47 of 300
47. Question
In a patient who has mitral valve insufficiency, which procedure does not require prophylactic antibiotic therapy?
Correct
Incorrect
-
Question 48 of 300
48. Question
In the ICU, a patient suddenly becomes unresponsive, pulseless, and hypotensive, with cardiac monitor indicating ventricular tachycardia. The crash cart is immediately available. The first therapeutic step among the following should be
Correct
Incorrect
-
Question 49 of 300
49. Question
A 60-year-old female with a history of urinary tract infection, steroiddependent chronic obstructive lung disease, and asthma presents with bilateral infiltrates and an eosinophil count of 15%. The least likely diagnosis is
Correct
Incorrect
-
Question 50 of 300
50. Question
A 50-year-old female is 5 ft 7 in. tall and weighs 165 lb. There is a family history of diabetes mellitus. Fasting blood glucose is 150 mg/dL on two occasions. She is asymptomatic, and physical exam shows no abnormalities. The treatment of choice is
Correct
Incorrect
-
Question 51 of 300
51. Question
Autoimmune thyroiditis can be confirmed in this patient by
Correct
Incorrect
-
Question 52 of 300
52. Question
The treatment of choice for Pagets disease patient is
Correct
Incorrect
-
Question 53 of 300
53. Question
In the advanced stage of addisonâs disease, the most likely electrolyte abnormalities will be
Correct
Incorrect
-
Question 54 of 300
54. Question
A 35-year-old alcoholic male is admitted for nausea, vomiting, and abdominal pain that radiates to the back. The laboratory value that suggests a poor prognosis in this patient is
Correct
Incorrect
-
Question 55 of 300
55. Question
The best way to eradicate H. pylori in this patient is
Correct
Incorrect
-
Question 56 of 300
56. Question
A 23-year-old man develops iron-deficiency anemia and hemepositive stools. His weight is stable. A few telangiectasias are present on the lips. Abdominal exam is negative without hepatosplenomegaly
Correct
Incorrect
-
Question 57 of 300
57. Question
Which is the most useful first step in the assessment of hyponatremia in a patient of Inappropriate secretion of antidiuretic hormone patient?
Correct
Incorrect
-
Question 58 of 300
58. Question
A chronic lymphatic leukemia patient will require chemotherapy
Correct
Incorrect
-
Question 59 of 300
59. Question
Many physiologic changes are associated with aging. Which of the following physiologic parameters does not change with age?
Correct
Incorrect
-
Question 60 of 300
60. Question
Pisiform is considered as a sesamoid bone in the tendon of?
Correct
Incorrect
-
Question 61 of 300
61. Question
Which is not a part of the hypogastric sheath?
Correct
Incorrect
-
Question 62 of 300
62. Question
Supination of forearm with the elbow extended is brought about mainly by?
Correct
Supinators of the forearm are:
Biceps brachii (Involved in rapid supination with elbow flexed and supination against resistance.)
Supinator (Mainly concerned with slow supination with elbow extended.)
Ref: B.D. Chaurasiaâs Human Anatomy, 4th Edition, Vol 1, p151.Incorrect
-
Question 63 of 300
63. Question
Anterior choroidal artery is a branch of?
Correct
Anterior choroidal artery arises from the internal carotid artery.
Anterior cerebral artery is a branch of internal carotid artery.
Posterior cerebral artery is a branch of vertebral artery.
Anterior communicating artery is a branch of anterior cerebral artery.Incorrect
-
Question 64 of 300
64. Question
Muscles supplied by anterior division of mandibular nerve are all except?
Correct
Mandibular nerve can be divided into 3 parts â main trunk, anterior division and posterior division.
Muscles supplied by main trunk:
Tensor veli palatini
Tensor tympani
Medial pterygoid
Muscles supplied by anterior division:
Lateral pterygoid
Temporalis
Masseter
Muscles supplied by posterior division:
Anterior belly of digastric
MylohyoidIncorrect
-
Question 65 of 300
65. Question
Toynbeeâs muscle is?
Correct
Incorrect
-
Question 66 of 300
66. Question
The Couinaudâs segmental nomenclature is based on the position of the
Correct
Couinaudâs segmental nomenclature is based on position of hepatic veins and portal vein. It is the French system.
Incorrect
-
Question 67 of 300
67. Question
All of the following are branches of the external carotid artery except:
Correct
Anterior ethmoidal artery arises from ophthalmic branch of internal carotid artery.
Incorrect
-
Question 68 of 300
68. Question
Retinal isomerase catalyses the conversion of?
Correct
- When light energy is absorbed by rhodopsin, photoactivation of electrons in the retinal portion of rhodopsin leads to the change of cis form of retinal into the all trans form.
- The first stage in the reformation of rhodopsin is to reconvert the all trans retinal into 11 cis retinal.
- This process is catalysed by the enzyme â retinal isomerase.
Ref: Textbook of Medical Physiology, Guyton and Hall, 11th edition, page 629.
Incorrect
- When light energy is absorbed by rhodopsin, photoactivation of electrons in the retinal portion of rhodopsin leads to the change of cis form of retinal into the all trans form.
- The first stage in the reformation of rhodopsin is to reconvert the all trans retinal into 11 cis retinal.
- This process is catalysed by the enzyme â retinal isomerase.
Ref: Textbook of Medical Physiology, Guyton and Hall, 11th edition, page 629.
-
Question 69 of 300
69. Question
Which is a true statement regarding inhibin?
Correct
- Inhibin is secreted by the sertoli cells in males and by the granulosa cells in females.
- It inhibits the secretion of Gonadotrophin Releasing Hormone.
- It is a glycoprotein with molecular weight between 10,000 and 30,000.
- It has potent inhibitory feedback effect on the anterior pituitary gland which in turn provides negative feedback mechanism for the control of spermatogenesis.
Ref: Textbook of Medical Physiology, Guyton and Hall, 11th edition, page 1007.
Incorrect
- Inhibin is secreted by the sertoli cells in males and by the granulosa cells in females.
- It inhibits the secretion of Gonadotrophin Releasing Hormone.
- It is a glycoprotein with molecular weight between 10,000 and 30,000.
- It has potent inhibitory feedback effect on the anterior pituitary gland which in turn provides negative feedback mechanism for the control of spermatogenesis.
Ref: Textbook of Medical Physiology, Guyton and Hall, 11th edition, page 1007.
-
Question 70 of 300
70. Question
Regarding organ of Corti, all are true except?
Correct
- Hensenâs cells are supporting cells of organ of Corti which lie outside the outer hair cells.
- Reticular lamina is a tough membrane which is supported by rods of Corti.
- Tectorial membrane is a thin, gelatinous, elastic structure made of glycoprotein.
- Efferent cholinergic fibers arise from ipsilateral and contralateral superior olivary nucleus.
Ref: Textbook of Physiology, A K Jain, 3rd edition, Volume 2, page 1103.
Incorrect
- Hensenâs cells are supporting cells of organ of Corti which lie outside the outer hair cells.
- Reticular lamina is a tough membrane which is supported by rods of Corti.
- Tectorial membrane is a thin, gelatinous, elastic structure made of glycoprotein.
- Efferent cholinergic fibers arise from ipsilateral and contralateral superior olivary nucleus.
Ref: Textbook of Physiology, A K Jain, 3rd edition, Volume 2, page 1103.
-
Question 71 of 300
71. Question
Juxtamedullary nephrons:
Correct
- Juxtamedullary nephrons comprise 14-15% nephrons in the kidney.
- They have large sized glomeruli which are situated at the junction of the cortex and medulla of the kidney.
- The rate of filtration is high.
- They have long loops of Henle which penetrate into the medulla.
- Their Henle loops manifest vascular supply in the form of vasa recta.
- Both descending and ascending limbs of loop of Henle contain thin segments. (In cortical nephrons, descending limb contains a thin segment whereas the ascending limb contains a thick segment.)
Incorrect
- Juxtamedullary nephrons comprise 14-15% nephrons in the kidney.
- They have large sized glomeruli which are situated at the junction of the cortex and medulla of the kidney.
- The rate of filtration is high.
- They have long loops of Henle which penetrate into the medulla.
- Their Henle loops manifest vascular supply in the form of vasa recta.
- Both descending and ascending limbs of loop of Henle contain thin segments. (In cortical nephrons, descending limb contains a thin segment whereas the ascending limb contains a thick segment.)
-
Question 72 of 300
72. Question
Regarding somatomedin, which is true?
Correct
- Somatomedin increases glucose oxidation in fat.
- Activity of somatomedin is reduced by protein deficiency and glucocorticoids.
- It has insulin like effect on tissues and causes lipolysis and increased glucose and amino acid transport by muscle.
Ref: Textbook of Physiology, A K Jain, 3rd edition, Volume 2, page 670.
Incorrect
- Somatomedin increases glucose oxidation in fat.
- Activity of somatomedin is reduced by protein deficiency and glucocorticoids.
- It has insulin like effect on tissues and causes lipolysis and increased glucose and amino acid transport by muscle.
Ref: Textbook of Physiology, A K Jain, 3rd edition, Volume 2, page 670.
-
Question 73 of 300
73. Question
Primary hyperaldosteronism is characterised by?
Correct
- Primary hyperaldosteronism is characterised by absence of peripheral oedema, metabolic alkalosis and marked fall in plasma potassium levels.
- There is rise in plasma sodium, but decrease in sodium content of sweat, saliva and GI secretions.
Incorrect
- Primary hyperaldosteronism is characterised by absence of peripheral oedema, metabolic alkalosis and marked fall in plasma potassium levels.
- There is rise in plasma sodium, but decrease in sodium content of sweat, saliva and GI secretions.
-
Question 74 of 300
74. Question
Cycle which is absent in humans is?
Correct
DIFFERENCE BETWEEN ESTROUS CYCLE AND MENSTRUAL CYCLE
- Estrous cycle in seen in non primate mammals, whereas menstrual cycle is seen in higher primates like humans.
- If conception does not occur, endometrium is reabsorbed in estrous cycle, whereas endometrium is shed in menstrual cycle.
- Females are sexually receptive only near the time of ovulation in estrous cycle. In menstrual cycle, it does not depend on the time of ovulation.Menstrual cycle in human females
COMPONENTS OF MENSTRUAL CYCLE
- It has 3 components â Ovarian cycle, Uterine cycle (endometrial cycle) and Hormonal cycle.
- The ovarian cycle consists of 3 phases â Follicular phase, Ovulation and Luteal phase.
- The uterine cycle consists of 3 phases â Menstrual phase, Proliferative phase and Secretory phase.
Image credits : Wikipedia
Incorrect
DIFFERENCE BETWEEN ESTROUS CYCLE AND MENSTRUAL CYCLE
- Estrous cycle in seen in non primate mammals, whereas menstrual cycle is seen in higher primates like humans.
- If conception does not occur, endometrium is reabsorbed in estrous cycle, whereas endometrium is shed in menstrual cycle.
- Females are sexually receptive only near the time of ovulation in estrous cycle. In menstrual cycle, it does not depend on the time of ovulation.Menstrual cycle in human females
COMPONENTS OF MENSTRUAL CYCLE
- It has 3 components â Ovarian cycle, Uterine cycle (endometrial cycle) and Hormonal cycle.
- The ovarian cycle consists of 3 phases â Follicular phase, Ovulation and Luteal phase.
- The uterine cycle consists of 3 phases â Menstrual phase, Proliferative phase and Secretory phase.
Image credits : Wikipedia
-
Question 75 of 300
75. Question
Which of the following are vitamin K dependant clotting factors?
Correct
Vitamin K dependant clotting factors are II, VII, IX and X. Vitamin K is required for the gamma carboxylation of glutamic acid residues.
Incorrect
Vitamin K dependant clotting factors are II, VII, IX and X. Vitamin K is required for the gamma carboxylation of glutamic acid residues.
-
Question 76 of 300
76. Question
Which of the following inhibits thrombin activity?
Correct
Thrombin inhibitors in plasma are:
- Antithrombin III (75% of total activity)
- Alpha 2 macroglobulin
- Heparin cofactor IIÂ (minor)
- Alpha 1 antitrypsin (minor)
So both A and D are correct, but the best answer is D (Alpha 2 macroglobulin).
Incorrect
Thrombin inhibitors in plasma are:
- Antithrombin III (75% of total activity)
- Alpha 2 macroglobulin
- Heparin cofactor IIÂ (minor)
- Alpha 1 antitrypsin (minor)
So both A and D are correct, but the best answer is D (Alpha 2 macroglobulin).
-
Question 77 of 300
77. Question
A newly posted junior doctor had difficulty in finding out base deficit/excess for blood in a given patient. An experienced senior resident advised a quick method to determine acid base composition of blood based on PCO2. Which of the following is the likely method he suggested to predict acid base composition of blood?
Correct
Siggard Andersen normogram helps to predict the acid base status of blood. PCO2 is plotted in the Y axis and pH in the X axis.
Incorrect
Siggard Andersen normogram helps to predict the acid base status of blood. PCO2 is plotted in the Y axis and pH in the X axis.
-
Question 78 of 300
78. Question
The force of muscle contraction can be increased by all of the following except :
Correct
Action potential is an âall or noneâ phenomenon. Increasing the amplitude beyond the threshold level does not influence the outcome.
Incorrect
Action potential is an âall or noneâ phenomenon. Increasing the amplitude beyond the threshold level does not influence the outcome.
-
Question 79 of 300
79. Question
The main excitatory neurotransmitter in the CNS is:
Correct
GABA is the main inhibitory neurotransmitter.
Excitatory neurotransmitters â Glutamate, Aspartate
Inhibitory neurotransmitters â Glycine, GABA (Gamma Amino Butyric Acid)Incorrect
GABA is the main inhibitory neurotransmitter.
Excitatory neurotransmitters â Glutamate, Aspartate
Inhibitory neurotransmitters â Glycine, GABA (Gamma Amino Butyric Acid) -
Question 80 of 300
80. Question
Before the onset of puberty, the GnRH neurons are under the inhibitory control of :
Correct
Incorrect
-
Question 81 of 300
81. Question
Sertoli cells have receptors for:
Correct
Incorrect
-
Question 82 of 300
82. Question
During the cardiac cycle the opening of the aortic valve takes place at the :
Correct
Valvular events in cardiac cycle:
- Atrio-ventricular valves close at the end of diastole
- Semilunar valves open at the end of isovolumetric contraction phase
- Semilunar valves close at the end of systole
- Atrio-ventricular valves open at the end of isovolumetric relaxation phase
Incorrect
Valvular events in cardiac cycle:
- Atrio-ventricular valves close at the end of diastole
- Semilunar valves open at the end of isovolumetric contraction phase
- Semilunar valves close at the end of systole
- Atrio-ventricular valves open at the end of isovolumetric relaxation phase
-
Question 83 of 300
83. Question
Abdominal approach is used in?
Correct
- Ripsteinâs operation is the abdominal approach for rectal prolapse.
- The abdominal approach is recommended in patients with complete rectal prolapse who are otherwise in good health.
- In Ripsteinâs operation, the rectosigmoid junction is hitched up to the front of the sacrum using a teflon sling.
Ref: Bailey and Loveâs Short Practice of Surgery, 25th edition, p 1225.
Incorrect
- Ripsteinâs operation is the abdominal approach for rectal prolapse.
- The abdominal approach is recommended in patients with complete rectal prolapse who are otherwise in good health.
- In Ripsteinâs operation, the rectosigmoid junction is hitched up to the front of the sacrum using a teflon sling.
Ref: Bailey and Loveâs Short Practice of Surgery, 25th edition, p 1225.
-
Question 84 of 300
84. Question
Most common type of teratoma of the testis is?
Correct
- Teratoma arises from totipotent cells in the rete testis.
- Histologically it is classified into 4 types:
- Teratoma differentiated
- Malignant teratoma intermediate
- Malignant teratoma anaplastic
- Malignant teratoma trophoblastic
- The most common type is malignant teratoma intermediate.
- It contains malignant and incompletely differentiated components.
Ref: Bailey and Loveâs Short Practice of Surgery, 25th edition, p 1385.
Incorrect
- Teratoma arises from totipotent cells in the rete testis.
- Histologically it is classified into 4 types:
- Teratoma differentiated
- Malignant teratoma intermediate
- Malignant teratoma anaplastic
- Malignant teratoma trophoblastic
- The most common type is malignant teratoma intermediate.
- It contains malignant and incompletely differentiated components.
Ref: Bailey and Loveâs Short Practice of Surgery, 25th edition, p 1385.
-
Question 85 of 300
85. Question
Which of the following is true regarding hereditary pancreatitis?
Correct
- Hereditary pancreatitis is an autosomal dominant disorder.
- It has 80% penetrance.
- It is associated with mutations in the cationic trypsinogen gene located on chromosome 7.
- IgG4 concentrations are elevated in autoimmune pancreatitis.
Ref: Bailey and Loveâs Short Practice of Surgery, 25th edition, p 1146.
Incorrect
- Hereditary pancreatitis is an autosomal dominant disorder.
- It has 80% penetrance.
- It is associated with mutations in the cationic trypsinogen gene located on chromosome 7.
- IgG4 concentrations are elevated in autoimmune pancreatitis.
Ref: Bailey and Loveâs Short Practice of Surgery, 25th edition, p 1146.
-
Question 86 of 300
86. Question
Stemmerâs sign is due to?Correct
- Stemmerâs sign is seen in lymphoedema.
- Skin on the dorsum of the toes cannot be pinched due to subcutaneous fibrosis.
- This is called Stemmerâs sign.
Ref: Bailey and Loveâs Short Practice of Surgery, 25th edition, p 947.
Incorrect
- Stemmerâs sign is seen in lymphoedema.
- Skin on the dorsum of the toes cannot be pinched due to subcutaneous fibrosis.
- This is called Stemmerâs sign.
Ref: Bailey and Loveâs Short Practice of Surgery, 25th edition, p 947.
-
Question 87 of 300
87. Question
All are true regarding Caroliâs disease except?
Correct
- Caroliâs disease is congenital dilatation of the intrahepatic biliary tree.
- This disease is complicated by presence of intrahepatic stone formation.
- Patients can present with abdominal pain or sepsis.
- Biliary stasis with stone formation can lead to biliary sepsis.
- Malignant change in the ductal system can lead to cholangiocarcinoma.
Ref: Bailey and Loveâs Short Practice of Surgery, 25th edition, p 1093.
Incorrect
- Caroliâs disease is congenital dilatation of the intrahepatic biliary tree.
- This disease is complicated by presence of intrahepatic stone formation.
- Patients can present with abdominal pain or sepsis.
- Biliary stasis with stone formation can lead to biliary sepsis.
- Malignant change in the ductal system can lead to cholangiocarcinoma.
Ref: Bailey and Loveâs Short Practice of Surgery, 25th edition, p 1093.
-
Question 88 of 300
88. Question
Which of the following is true regarding type III neuroendocrine tumour of the stomach?
Correct
- Neuroendocrine tumours of the stomach are rare tumours which comprise about 5% of all neuroendocrine tumours of the gastrointestinal tract.
- The most frequent neuroendocrine tumour of the stomach is type I tumour.
- Type III tumours are low grade malignant differentiated tumours.
- Serum gastrin is normal.
- The common symptom is upper gastrointestinal bleeding.
- Gastrectomy, lymph node resection and resection of liver metastases is the treatment of choice.
Ref: Bailey and Loveâs Short Practice of Surgery, 25th edition, p 821.
Incorrect
- Neuroendocrine tumours of the stomach are rare tumours which comprise about 5% of all neuroendocrine tumours of the gastrointestinal tract.
- The most frequent neuroendocrine tumour of the stomach is type I tumour.
- Type III tumours are low grade malignant differentiated tumours.
- Serum gastrin is normal.
- The common symptom is upper gastrointestinal bleeding.
- Gastrectomy, lymph node resection and resection of liver metastases is the treatment of choice.
Ref: Bailey and Loveâs Short Practice of Surgery, 25th edition, p 821.
-
Question 89 of 300
89. Question
Delayed obstruction of urinary tract after TURP is most commonly caused by?
Correct
Most common cause of delayed obstruction of urinary tract after Transurethral Resection of Prostate (TURP) is bladder neck stenosis.
Late complications of TURP include:
- Urethral stricture (3.8%)
- Bladder neck stenosis (4%)
- Stress incontinence (2.2%)
- Sexual dysfunction: retrograde ejaculation (65-70%) and erectile dysfunction (6%).
- Penoscrotal stricture, Bulbar stricture, Membranous urethral stricture and Submeatal strictures are subtypes of urethral strictures.
- Urethral stricture can be treated by end to end urethroplasty.
- Bladder neck stenosis can be corrected by endoscopic incision.
References:Â
- Manual Endourology: Training for Residents By Rudolf Hohenfellner, Jens-Uwe Stolzenburg
- Post-TURP obliterative urethral stricture: Unusual treatment and favourable result. African Journal of Urology; A. Bhageria, , B. Nayak, P.K. Rai, P.N. Dogra
Incorrect
Most common cause of delayed obstruction of urinary tract after Transurethral Resection of Prostate (TURP) is bladder neck stenosis.
Late complications of TURP include:
- Urethral stricture (3.8%)
- Bladder neck stenosis (4%)
- Stress incontinence (2.2%)
- Sexual dysfunction: retrograde ejaculation (65-70%) and erectile dysfunction (6%).
- Penoscrotal stricture, Bulbar stricture, Membranous urethral stricture and Submeatal strictures are subtypes of urethral strictures.
- Urethral stricture can be treated by end to end urethroplasty.
- Bladder neck stenosis can be corrected by endoscopic incision.
References:Â
- Manual Endourology: Training for Residents By Rudolf Hohenfellner, Jens-Uwe Stolzenburg
- Post-TURP obliterative urethral stricture: Unusual treatment and favourable result. African Journal of Urology; A. Bhageria, , B. Nayak, P.K. Rai, P.N. Dogra
-
Question 90 of 300
90. Question
Areas involved in posterior cerebral artery infarct are all except?
Correct
Blood supply of brain â Anterior cerebral artery (blue), middle cerebral artery (red) &Â posterior cerebral artery (yellow).
AREAS INVOLVED IN POSTERIOR CEREBRAL ARTERY INFARCT
- Occipital lobe
- Infero-medial temporal lobe
- Postero-inferior parietal lobe
- Large parts of thalamus
- Midbrain
- Posterior part of choroid plexus
Incorrect
Blood supply of brain â Anterior cerebral artery (blue), middle cerebral artery (red) &Â posterior cerebral artery (yellow).
AREAS INVOLVED IN POSTERIOR CEREBRAL ARTERY INFARCT
- Occipital lobe
- Infero-medial temporal lobe
- Postero-inferior parietal lobe
- Large parts of thalamus
- Midbrain
- Posterior part of choroid plexus
-
Question 91 of 300
91. Question
A patient presents to the emergency department with an abdominal trauma with signs of shock and peritonitis. Airway and breathing were checked. 2 large bore cannulas were inserted to secure IV access. What is the next step in the management of this patient?
Correct
MANAGEMENT OF A CASE OF ABDOMINAL TRAUMA
- Maintenance of airway, breathing and circulation is the first priority
- The patient should be intubated in case of airway block
- If the breathing is compromised, the patient should be ventilated with a high fraction of inspired oxygen
- Large bore IV cannulas should be inserted and IV fluids should be administered
- In case of external bleeding, direct pressure should be applied to control it
- Spinal immobilisation should be done in cases with suspected spinal cord injury
Indications for laparotomy in blunt trauma abdomen
- Hemodynamically unstable patients with shock
- Signs of peritonitis
- Progressive deterioration of the patientâs status
- Identification of hemoperitoneum after Focused Assessment with Sonography in Trauma (FAST) or Diagnostic Peritoneal Lavage (DPL)
Non surgical management
- If the patent is hemodynamically stable, further investigations like CT scan can be done to visualise the site of injury
Incorrect
MANAGEMENT OF A CASE OF ABDOMINAL TRAUMA
- Maintenance of airway, breathing and circulation is the first priority
- The patient should be intubated in case of airway block
- If the breathing is compromised, the patient should be ventilated with a high fraction of inspired oxygen
- Large bore IV cannulas should be inserted and IV fluids should be administered
- In case of external bleeding, direct pressure should be applied to control it
- Spinal immobilisation should be done in cases with suspected spinal cord injury
Indications for laparotomy in blunt trauma abdomen
- Hemodynamically unstable patients with shock
- Signs of peritonitis
- Progressive deterioration of the patientâs status
- Identification of hemoperitoneum after Focused Assessment with Sonography in Trauma (FAST) or Diagnostic Peritoneal Lavage (DPL)
Non surgical management
- If the patent is hemodynamically stable, further investigations like CT scan can be done to visualise the site of injury
-
Question 92 of 300
92. Question
Which of the following is not a component of Total Parenteral Nutrition?
Correct
You are here: Home » AIPGMEE » AIPGMEE 2011 » .
TOTAL PARENTERAL NUTRITION â SURGERY MCQ
Which of the following is not a component of Total Parenteral Nutrition?
A. Amino acids
B. Fats
C. Vitamins
D. FiberCorrect answer : D. Fiber
COMPONENTS OF TOTAL PARENTERAL NUTRITION
- Total parenteral nutrition should provide both energy and other essential nutrients.
- Carbohydrates, lipids and amino acids provide energy.
- The other essential nutrients to be included are: minerals, vitamins and water.
- Dietary fiber is required only in case of enteral nutrition. It is not given as a part of parenteral nutrition.
Related reading: ESPEN (The European Society for Clinical Nutrition and Metabolism):Â Guidelines on Parenteral Nutrition: Intensive care
Incorrect
You are here: Home » AIPGMEE » AIPGMEE 2011 » .
TOTAL PARENTERAL NUTRITION â SURGERY MCQ
Which of the following is not a component of Total Parenteral Nutrition?
A. Amino acids
B. Fats
C. Vitamins
D. FiberCorrect answer : D. Fiber
COMPONENTS OF TOTAL PARENTERAL NUTRITION
- Total parenteral nutrition should provide both energy and other essential nutrients.
- Carbohydrates, lipids and amino acids provide energy.
- The other essential nutrients to be included are: minerals, vitamins and water.
- Dietary fiber is required only in case of enteral nutrition. It is not given as a part of parenteral nutrition.
Related reading: ESPEN (The European Society for Clinical Nutrition and Metabolism):Â Guidelines on Parenteral Nutrition: Intensive care
-
Question 93 of 300
93. Question
Most common splenic cyst is?
Correct
Incorrect
-
Question 94 of 300
94. Question
Most common splenic cyst is?
Correct
Incorrect
-
Question 95 of 300
95. Question
Most common splenic cyst is?
Correct
Incorrect
-
Question 96 of 300
96. Question
Surgery for correction of cryptorchidism is best done before?
Correct
Surgical orchidopexy should be done if spontaneous descent has not occurred by 1 year of age.
Incorrect
Surgical orchidopexy should be done if spontaneous descent has not occurred by 1 year of age.
-
Question 97 of 300
97. Question
The commonest cause of abdominal aortic aneurysm is?
Correct
90% of all abdominal aortic aneurysms greater than 4 cm are caused by atherosclerotic disease.
Incorrect
90% of all abdominal aortic aneurysms greater than 4 cm are caused by atherosclerotic disease.
-
Question 98 of 300
98. Question
Lordâs plication is used in the treatment of ?
Correct
- Lordâs plication is a surgical procedure used for the treatment of hydrocele
- The hydrocele fluid is drained and the redundant sac is plicated
- It is used for treatment of small and medium sized hydroceles
Incorrect
- Lordâs plication is a surgical procedure used for the treatment of hydrocele
- The hydrocele fluid is drained and the redundant sac is plicated
- It is used for treatment of small and medium sized hydroceles
-
Question 99 of 300
99. Question
Which of the following conditions is associated with multiple cutaneous sebaceous adenomas?
Correct
Incorrect
-
Question 100 of 300
100. Question
The earliest manifestation of increased intracranial pressure following head injury is:
Correct
Incorrect
-
Question 101 of 300
101. Question
The following statements regarding finasteride are true except:
Correct
Incorrect
-
Question 102 of 300
102. Question
Sclerotherapy is ideal for treatment of
Correct
Incorrect
-
Question 103 of 300
103. Question
All of the following are true for patients of ulcerative colitis associated with primary sclerosing cholangitis (PSC), except:
Correct
Incorrect
-
Question 104 of 300
104. Question
Gardnerâs syndrome is a rare hereditary disorder involving the colon. It is characterized by:
Correct
Incorrect
-
Question 105 of 300
105. Question
The most preferred approach for pituitary surgery at the present time is:
Correct
Incorrect
-
Question 106 of 300
106. Question
The most common complication seen in hiatus hernia is:
Correct
Incorrect
-
Question 107 of 300
107. Question
Lumbar sympathectomy is of value in the management of:
Correct
The sympathetic system controls the blood flow to skin (not muscle). Sympathetic tone has a vasoconstrictor effect. Hence lumbar sympathectomy will cause vasodilation and increased blood flow to skin of toes.
Incorrect
The sympathetic system controls the blood flow to skin (not muscle). Sympathetic tone has a vasoconstrictor effect. Hence lumbar sympathectomy will cause vasodilation and increased blood flow to skin of toes.
-
Question 108 of 300
108. Question
A blood stained discharge from the nipple indicates one of the following:
Correct
Causes of blood stained nipple discharge
- Intraductal papilloma
- Carcinoma
- Duct ectasia
- Fibrocystic disease
Incorrect
Causes of blood stained nipple discharge
- Intraductal papilloma
- Carcinoma
- Duct ectasia
- Fibrocystic disease
-
Question 109 of 300
109. Question
Lyre sign in seen in?
Correct
- Lyre sign is an angiographic finding seen in carotid body tumour of the neck.
- There is splaying apart of internal and external carotid arteries on angiogram.
Ref: Diseases of Ear, Nose and Throat by P.L. Dhingra, 5th edition, p460.
Incorrect
- Lyre sign is an angiographic finding seen in carotid body tumour of the neck.
- There is splaying apart of internal and external carotid arteries on angiogram.
Ref: Diseases of Ear, Nose and Throat by P.L. Dhingra, 5th edition, p460.
-
Question 110 of 300
110. Question
Hitzelbergerâs sign is?
Correct
- Hitzelbergerâs sign is seen in acoustic neuroma.
- It occurs due to involvement of the 7th cranial nerve.
- There is early involvement of the sensory fibers which causes hypoesthesia of the posterior meatal wall.
- Paraesthesia of face and reduced corneal sensitivity occurs due to involvement of 5th cranial nerve.
Ref: Diseases of Ear, Nose and Throat by P.L. Dhingra, 5th edition, p124, 125.
Incorrect
- Hitzelbergerâs sign is seen in acoustic neuroma.
- It occurs due to involvement of the 7th cranial nerve.
- There is early involvement of the sensory fibers which causes hypoesthesia of the posterior meatal wall.
- Paraesthesia of face and reduced corneal sensitivity occurs due to involvement of 5th cranial nerve.
Ref: Diseases of Ear, Nose and Throat by P.L. Dhingra, 5th edition, p124, 125.
-
Question 111 of 300
111. Question
Orbital apex syndrome involves?
Correct
- Orbital apex syndrome is a complication of paranasal sinus infection.
- Superior orbital fissure syndrome along with involvement of maxillary division of trigeminal nerve and optic nerve is known as orbital apex syndrome.
- In superior orbital fissure syndrome, there is progressive paralysis of abducent, oculomotor and trochlear nerves.
Ref: Diseases of Ear, Nose and Throat by P.L. Dhingra, 5th edition, p213.
Image credits:Â https://en.wikipedia.org/wiki/File:Gray777.pngIncorrect
- Orbital apex syndrome is a complication of paranasal sinus infection.
- Superior orbital fissure syndrome along with involvement of maxillary division of trigeminal nerve and optic nerve is known as orbital apex syndrome.
- In superior orbital fissure syndrome, there is progressive paralysis of abducent, oculomotor and trochlear nerves.
Ref: Diseases of Ear, Nose and Throat by P.L. Dhingra, 5th edition, p213.
Image credits:Â https://en.wikipedia.org/wiki/File:Gray777.png -
Question 112 of 300
112. Question
Pain in tonsillar fossa and upper neck is characteristic of?
Correct
- Eagleâs syndrome is characterised by pain in tonsillar fossa and upper neck.
- Pain radiates of ipsilateral ear.
- There is aggravation of pain on swallowing.
- It is caused by calcification of stylohyoid ligament / elongation of styloid process.
Ref: Diseases of Ear, Nose and Throat by P.L. Dhingra, 5th edition, p287.
Incorrect
- Eagleâs syndrome is characterised by pain in tonsillar fossa and upper neck.
- Pain radiates of ipsilateral ear.
- There is aggravation of pain on swallowing.
- It is caused by calcification of stylohyoid ligament / elongation of styloid process.
Ref: Diseases of Ear, Nose and Throat by P.L. Dhingra, 5th edition, p287.
-
Question 113 of 300
113. Question
The following is true for Gradenigoâs syndrome?
Correct
- Gradenigoâs syndrome is seen in petrositis.
- It is characterised by lateral rectus palsy, persistent ear discharge and deep seated ear or retro-orbital pain.
- Mastoid tenderness, hearing loss and sagging of posterosuperior meatal wall are seen acute mastoiditis.
Ref: Diseases of Ear, Nose and Throat by P.L. Dhingra, 5th edition, p86, 89.
Incorrect
- Gradenigoâs syndrome is seen in petrositis.
- It is characterised by lateral rectus palsy, persistent ear discharge and deep seated ear or retro-orbital pain.
- Mastoid tenderness, hearing loss and sagging of posterosuperior meatal wall are seen acute mastoiditis.
Ref: Diseases of Ear, Nose and Throat by P.L. Dhingra, 5th edition, p86, 89.
-
Question 114 of 300
114. Question
Triad of Klippel Feil syndrome consists of all except?
Correct
Elevated scapula can be seen as a part of Klippel Feil syndrome. But it does not constitute the characteristic triad.
Incorrect
Elevated scapula can be seen as a part of Klippel Feil syndrome. But it does not constitute the characteristic triad.
-
Question 115 of 300
115. Question
Brown tumour is characteristic of?
Correct
Brown tumour occurs due to the increased osteoclastic activity in hyperparathyroidism. It is a benign mass of reactive tissue.
Incorrect
Brown tumour occurs due to the increased osteoclastic activity in hyperparathyroidism. It is a benign mass of reactive tissue.
-
Question 116 of 300
116. Question
Clavicular fracture is treated by?
Correct
- Clavicular fracture heals with good functional results even if there is moderate displacement of the fracture ends
- Perfect reduction is not essential
- Treatment consists of a âFigure of Eightâ bandage which is periodically tightened
- A Â sling can be used initially to support the upper limb
- In case of elderly patients with risk of axillary neurovascular compression, only a sling is used instead of the âfigure of eightâ bandage
- Open reduction is done in case of non union, neurovascular damage, floating shoulder and wide separation of the fracture ends with interposition of soft tissue.
Incorrect
- Clavicular fracture heals with good functional results even if there is moderate displacement of the fracture ends
- Perfect reduction is not essential
- Treatment consists of a âFigure of Eightâ bandage which is periodically tightened
- A Â sling can be used initially to support the upper limb
- In case of elderly patients with risk of axillary neurovascular compression, only a sling is used instead of the âfigure of eightâ bandage
- Open reduction is done in case of non union, neurovascular damage, floating shoulder and wide separation of the fracture ends with interposition of soft tissue.
-
Question 117 of 300
117. Question
True statement about involucrum and sequestrum:
Correct
Incorrect
-
Question 118 of 300
118. Question
Gallowâs traction is used for ?
Correct
Gallowâs traction is used in infants and children with femoral fractures.
Indications Gallows Traction
- Child must weigh less than 12 kg
- Femoral fractures
- Skin must be intact
Incorrect
Gallowâs traction is used in infants and children with femoral fractures.
Indications Gallows Traction
- Child must weigh less than 12 kg
- Femoral fractures
- Skin must be intact
-
Question 119 of 300
119. Question
Anaesthetic implication in Gaisbockâs syndrome is?
Correct
- Gaisbockâs syndrome is polycythemia due to reduction in plasma volume.
- It is usually seen in smokers, obese individuals and in patients with hypertension.
- Anaesthetic implications include cardiovascular problems and risk of deep vein thrombosis.
- May require venesection.
Incorrect
- Gaisbockâs syndrome is polycythemia due to reduction in plasma volume.
- It is usually seen in smokers, obese individuals and in patients with hypertension.
- Anaesthetic implications include cardiovascular problems and risk of deep vein thrombosis.
- May require venesection.
-
Question 120 of 300
120. Question
Guttmannâs sign is?
Correct
- Guttmannâs sign is stuffiness of the nostril which is a sign of successful stellate ganglion block.
- The other signs of successful stellate ganglion block include Hornerâs syndrome, engorged veins of the arm, flushing of the conjunctiva and sclera, increase in skin temperature and anhydrosis of the face and neck.
Ref: Leeâs Synopsis of Anaesthesia. 13th edition. p444.
Incorrect
- Guttmannâs sign is stuffiness of the nostril which is a sign of successful stellate ganglion block.
- The other signs of successful stellate ganglion block include Hornerâs syndrome, engorged veins of the arm, flushing of the conjunctiva and sclera, increase in skin temperature and anhydrosis of the face and neck.
Ref: Leeâs Synopsis of Anaesthesia. 13th edition. p444.
-
Question 121 of 300
121. Question
Thiopental is contraindicated in all of the following except?
Correct
- Thiopental is contraindicated in porphyria, status asthmaticus, pericardial tamponade, and severe shock.
- It causes centrally mediated respiratory depression.
- It can cause laryngeal spasm and bronchoconstriction in asthmatics.
- Thiopental causes depression of myocardial contractility.
- Peripheral vascular resistance falls and it can lead to hypotension, which can be exaggerated if there is hypovolemia.
Ref: Leeâs Synopsis of Anaesthesia. 13th edition. p154, 156.
Incorrect
- Thiopental is contraindicated in porphyria, status asthmaticus, pericardial tamponade, and severe shock.
- It causes centrally mediated respiratory depression.
- It can cause laryngeal spasm and bronchoconstriction in asthmatics.
- Thiopental causes depression of myocardial contractility.
- Peripheral vascular resistance falls and it can lead to hypotension, which can be exaggerated if there is hypovolemia.
Ref: Leeâs Synopsis of Anaesthesia. 13th edition. p154, 156.
-
Question 122 of 300
122. Question
Which of the following anaesthetic agents can induce epilepsy?
Correct
- Enflurane and Sevoflurane can induce epilepsy.
- The effect is much more pronounced with Enflurane.
- Tip to remember: Enflurane = Epilepsy (both start with E).
- Enflurane can also increase the intracranial pressure.
- Isoflurane can cause coronary steal syndrome.
- Methoxyflurane is nephrotoxic and can cause high output renal failure.
Incorrect
- Enflurane and Sevoflurane can induce epilepsy.
- The effect is much more pronounced with Enflurane.
- Tip to remember: Enflurane = Epilepsy (both start with E).
- Enflurane can also increase the intracranial pressure.
- Isoflurane can cause coronary steal syndrome.
- Methoxyflurane is nephrotoxic and can cause high output renal failure.
-
Question 123 of 300
123. Question
Anaesthetic agent contraindicated in hypertensives is?
Correct
ANAESTHETIC AGENT CONTRAINDICATED IN HYPERTENSIVES
- Ketamine increases heart rate, cardiac output and blood pressure
- These effects are due to stimulation of the sympathetic nervous system and decrease in reuptake of noradrenaline
- Hence it is contraindicated in:
- Hypertensives
- Ischemic  heart disease
- Congestive cardiac failure
- Arterial aneurysms
- It is also contraindicated in patients with increased intracranial tension
- It produces dissociative anaesthesia
Incorrect
ANAESTHETIC AGENT CONTRAINDICATED IN HYPERTENSIVES
- Ketamine increases heart rate, cardiac output and blood pressure
- These effects are due to stimulation of the sympathetic nervous system and decrease in reuptake of noradrenaline
- Hence it is contraindicated in:
- Hypertensives
- Ischemic  heart disease
- Congestive cardiac failure
- Arterial aneurysms
- It is also contraindicated in patients with increased intracranial tension
- It produces dissociative anaesthesia
-
Question 124 of 300
124. Question
Which anaesthetic agent is preferred in a patient taken for surgery with bilirubin of 8.6 mg/dl and serum creatinine of 2.1 mg/dl ?
Correct
- Atracurium is the preferred muscle relaxant in a patient with liver dysfunction / renal failure
- It undergoes non specific ester hydrolysis and non enzymatic degradation ( Hoffman degradation )
- It does not depend on the liver / kidney for its deactivation
- Other muscle relaxants that can be given in liver / renal disease are Cisatracurium and Mivacurium
Incorrect
- Atracurium is the preferred muscle relaxant in a patient with liver dysfunction / renal failure
- It undergoes non specific ester hydrolysis and non enzymatic degradation ( Hoffman degradation )
- It does not depend on the liver / kidney for its deactivation
- Other muscle relaxants that can be given in liver / renal disease are Cisatracurium and Mivacurium
-
Question 125 of 300
125. Question
Which one of the following is the fastest acting inhalational agent?
Correct
Sevoflurane is the fastest acting among the options given. Desflurane is THE fastest acting inhalational agent.
Incorrect
Sevoflurane is the fastest acting among the options given. Desflurane is THE fastest acting inhalational agent.
-
Question 126 of 300
126. Question
Which of the following anesthetic agents does not trigger malignant hyperthermia?
Correct
Anesthetic agents implicated in malignant hyperthermia:
- Succinyl choline
- Halothane
- Isoflurane
Incorrect
Anesthetic agents implicated in malignant hyperthermia:
- Succinyl choline
- Halothane
- Isoflurane
-
Question 127 of 300
127. Question
A 30 year old male is having Attic cholesteatoma of left ear with lateral sinus thrombophlebitis. Which of the following will be the operation of choice?
Correct
Attic cholesteatoma is an indication for modified radical mastoidectomy. (Canal wall down mastoidectomy)
Incorrect
Attic cholesteatoma is an indication for modified radical mastoidectomy. (Canal wall down mastoidectomy)
-
Question 128 of 300
128. Question
Which of the following is not a typical feature of Ménièreâs disease?
Correct
Features of Ménièreâs disease are:
- Aural fullness
- Fluctuating sensorineural deafness
- Fluctuating tinnitus
- Episodic vertigo
Incorrect
Features of Ménièreâs disease are:
- Aural fullness
- Fluctuating sensorineural deafness
- Fluctuating tinnitus
- Episodic vertigo
-
Question 129 of 300
129. Question
In addition to slow intravenous infusion, which of the following routes of administration allows for titration of the dose of a drug with the response:
Correct
Incorrect
-
Question 130 of 300
130. Question
Majority of drugs cross biological membranes primarily by:
Correct
Incorrect
-
Question 131 of 300
131. Question
Which of the following drugs acts by inhibiting an enzyme in the body:
Correct
Incorrect
-
Question 132 of 300
132. Question
The most appropriate route of administration for adrenaline in a case of anaphylactic shock is:
Correct
Incorrect
-
Question 133 of 300
133. Question
Intradermal drug sensitivity tests can detect the presence of following type of hypersensitivity:
Correct
Incorrect
-
Question 134 of 300
134. Question
âAddictionâ and âhabituationâ:
Correct
Incorrect
-
Question 135 of 300
135. Question
Which of the following organs is innervated only by parasympathetic nerves:
Correct
Incorrect
-
Question 136 of 300
136. Question
The smooth muscle structure that is relaxed by cholinergic drugs is:
A. Colon
B. Gastric fundus
C. Major bronchi
D. Bladder trigone
Correct
Incorrect
-
Question 137 of 300
137. Question
The following inhibitor binds only to the anionic site of the cholinesterase enzyme:
Correct
Incorrect
-
Question 138 of 300
138. Question
Beta adrenergic blockers lower intraocular tension by:
Correct
Incorrect
-
Question 139 of 300
139. Question
Atropine produces the following actions except:
Correct
Incorrect
-
Question 140 of 300
140. Question
The following type/types of noradrenaline uptake is blocked by reserpine:
Correct
Incorrect
-
Question 141 of 300
141. Question
The following is a selective α2 adrenoceptor antagonist:
Correct
Incorrect
-
Question 142 of 300
142. Question
The most important receptor involved in cytotoxic drug induced vomiting is:
Correct
Incorrect
-
Question 143 of 300
143. Question
Low doses of aspirin prolong bleeding time by selectively inhibiting synthesis of the following mediator in the platelets:
Correct
Incorrect
-
Question 144 of 300
144. Question
Select the drug which inhibits cyclooxygenase irreversibly:
Correct
Incorrect
-
Question 145 of 300
145. Question
Used as a remission inducing agent in rheumatoid arthritis, hydroxychloroquine:
Correct
Incorrect
-
Question 146 of 300
146. Question
Oxytocin is essential for:
Correct
Incorrect
-
Question 147 of 300
147. Question
Bisphosphonates are beneficial in the following conditions except:
Correct
Incorrect
-
Question 148 of 300
148. Question
Use of morphine in preanaesthetic medication:
Correct
Incorrect
-
Question 149 of 300
149. Question
Ventricular remodeling after myocardial infarction involves the mediation of:
Correct
Incorrect
-
Question 150 of 300
150. Question
Digitalis slows the heart in congestive heart failure by:
Correct
Incorrect
-
Question 151 of 300
151. Question
A 60 year old male presented with acute chest pain of 4 hours duration. Electrocardiographic examination revealed new Q wave with ST segment depression. He succumbed to his illness within 24 hours of admission. The heart revealed presence of a transmural haemorrhagic area over the septum and anterior wall of the left ventricle. Light microscopic examination is most likely to reveal:
Correct
After 24 hours of myocardial infarction, there is coagulative necrosis of myofibers with neutrophil infiltration.
Incorrect
After 24 hours of myocardial infarction, there is coagulative necrosis of myofibers with neutrophil infiltration.
-
Question 152 of 300
152. Question
All of the following statements about hairy cell leukaemia are true except:
Correct
Incorrect
-
Question 153 of 300
153. Question
Which is the most common cytogenetic abnormality in adult myelodysplastic syndrome (MDS)?
Correct
Incorrect
-
Question 154 of 300
154. Question
Which of the following is an automsomal dominant metabolic disorder?
Correct
Incorrect
-
Question 155 of 300
155. Question
Which of the following is an antiapoptotic gene?
Correct
Pro apoptotic genes â bax, bad, bcl-xS
Anti apoptotic genes â bcl-2, bcl-xLIncorrect
Pro apoptotic genes â bax, bad, bcl-xS
Anti apoptotic genes â bcl-2, bcl-xL -
Question 156 of 300
156. Question
A small calcium binding protein that modifies the activity of many enzymes and other proteins in response to changes of Ca2+ concentration, is known as:
Correct
Incorrect
-
Question 157 of 300
157. Question
Which of the following substances acts to increase the release of Ca2+ from endoplasmic reticulum ?
Correct
Inositol triphosphate (IP3) and diacylglycerol (DAG) are second messengers for G protein coupled receptors. IP3 stimulates release of calcium from endoplasmic reticulum. DAG activates protein kinase C.
Incorrect
Inositol triphosphate (IP3) and diacylglycerol (DAG) are second messengers for G protein coupled receptors. IP3 stimulates release of calcium from endoplasmic reticulum. DAG activates protein kinase C.
-
Question 158 of 300
158. Question
Which of the following can be a homologous substitution for valine in hemoglobin?
Correct
Valine can be replaced by glutamic acid, aspartic acid or alanine at position 67 of hemoglobin to produce Hb Milwaukee, Hb Bristol and Hb Sydney respectively. All of these are functionally normal hemoglobin types.
Incorrect
Valine can be replaced by glutamic acid, aspartic acid or alanine at position 67 of hemoglobin to produce Hb Milwaukee, Hb Bristol and Hb Sydney respectively. All of these are functionally normal hemoglobin types.
-
Question 159 of 300
159. Question
An enzyme that makes a double stranded DNA copy from a single stranded RNA template molecule is known as:
Correct
Reverse transcriptase is an RNA dependant DNA polymerase. It is seen in retro viruses
Incorrect
Reverse transcriptase is an RNA dependant DNA polymerase. It is seen in retro viruses
-
Question 160 of 300
160. Question
An enzyme that makes a double stranded DNA copy from a single stranded RNA template molecule is known as:
Correct
Reverse transcriptase is an RNA dependant DNA polymerase. It is seen in retro viruses
Incorrect
Reverse transcriptase is an RNA dependant DNA polymerase. It is seen in retro viruses
-
Question 161 of 300
161. Question
Which of the following is a membrane bound enzyme that catalyzes the formation of cyclic AMP from ATP?
Correct
Incorrect
-
Question 162 of 300
162. Question
Both Vitamin K and C are involved in:
Correct
Incorrect
-
Question 163 of 300
163. Question
Vitamin B12 acts as co-enzyme to which one of the following enzymes?
Correct
Incorrect
-
Question 164 of 300
164. Question
All of the following cell types contain the enzyme telomerase which protects the length of telomeres at the end of chromosomes, except:
Correct
Incorrect
-
Question 165 of 300
165. Question
All of the following enzymes are regulated by calcium or calmodulin, except:
Correct
Some enzymes regulated by calcium / calmodulin:
- Adenylate cyclase
- Guanylate cyclase
- Pyruvate kinase
- Pyruvate dehydrogenase
- Pyruvate carboxylase
- Phospholipase A2
- Glycogen synthase
- Glycerol 3 phosphate dehydrogenase
Incorrect
Some enzymes regulated by calcium / calmodulin:
- Adenylate cyclase
- Guanylate cyclase
- Pyruvate kinase
- Pyruvate dehydrogenase
- Pyruvate carboxylase
- Phospholipase A2
- Glycogen synthase
- Glycerol 3 phosphate dehydrogenase
-
Question 166 of 300
166. Question
Fluoride, used in the collection of blood samples for glucose estimation, inhibits the enzyme:
Correct
Incorrect
-
Question 167 of 300
167. Question
Which type of RNA has the highest percentage of modified base?
Correct
In tRNA, the bases (Adenine, Guanine, Uracil and Cytosine) are modified by methylation, reduction, deamination and rearrangement of glycosidic bonds.
Incorrect
In tRNA, the bases (Adenine, Guanine, Uracil and Cytosine) are modified by methylation, reduction, deamination and rearrangement of glycosidic bonds.
-
Question 168 of 300
168. Question
Mitochondrial DNA is:
Correct
Incorrect
-
Question 169 of 300
169. Question
Chymotrypsinogen is a
Correct
Incorrect
-
Question 170 of 300
170. Question
Anticoagulant used to estimate glucose from a sample sent from PHC is
Correct
Incorrect
-
Question 171 of 300
171. Question
Which of the following is true regarding Mittendorf dot?
Correct
- Mittendorf dot is a remnant of the anterior end of hyaloid artery.
- It is attached to the posterior lens capsule.
- It is associated with posterior polar cataract.
Ref: Comprehensive Ophthalmology, A. K. Khurana, 4th edition, p253.
Image credits:Â https://en.wikipedia.org/wiki/File:Schematic_diagram_of_the_human_eye_en.sIncorrect
- Mittendorf dot is a remnant of the anterior end of hyaloid artery.
- It is attached to the posterior lens capsule.
- It is associated with posterior polar cataract.
Ref: Comprehensive Ophthalmology, A. K. Khurana, 4th edition, p253.
Image credits:Â https://en.wikipedia.org/wiki/File:Schematic_diagram_of_the_human_eye_en.s -
Question 172 of 300
172. Question
Coganâs syndrome is associated with?
Correct
- Coganâs syndrome is seen in middle aged adults.
- The features include interstitial keratitis, acute tinnitus, vertigo and deafness.
- Treatment is by usage of topical and systemic corticosteroids.
- Early treatment is necessary to prevent permanent deafness and blindness.
Ref: Comprehensive Ophthalmology, A. K. Khurana, 4th edition, p114.
Incorrect
- Coganâs syndrome is seen in middle aged adults.
- The features include interstitial keratitis, acute tinnitus, vertigo and deafness.
- Treatment is by usage of topical and systemic corticosteroids.
- Early treatment is necessary to prevent permanent deafness and blindness.
Ref: Comprehensive Ophthalmology, A. K. Khurana, 4th edition, p114.
-
Question 173 of 300
173. Question
Stockerâs line is seen in?
Correct
- Pterygium is usually seen in elderly males and it presents as a triangular fold of conjunctiva which encroaches the cornea.
- It is usually seen on the nasal side.
- Stockerâs line is a deposition of iron seen in corneal epithelium anterior to advancing head of pterygium.
Incorrect
- Pterygium is usually seen in elderly males and it presents as a triangular fold of conjunctiva which encroaches the cornea.
- It is usually seen on the nasal side.
- Stockerâs line is a deposition of iron seen in corneal epithelium anterior to advancing head of pterygium.
-
Question 174 of 300
174. Question
Gunn sign is?
Correct
- Gunn sign is seen in grade III hypertensive retinopathy.
- The other findings in grade III hypertensive retinopathy include
- copper wiring of arterioles
- banking of veins distal to arteriovenous crossings
- right-angle deflection of veins
- flame shaped haemorrhages
- cotton wool spots
- hard exudates.
Incorrect
- Gunn sign is seen in grade III hypertensive retinopathy.
- The other findings in grade III hypertensive retinopathy include
- copper wiring of arterioles
- banking of veins distal to arteriovenous crossings
- right-angle deflection of veins
- flame shaped haemorrhages
- cotton wool spots
- hard exudates.
-
Question 175 of 300
175. Question
Bilateral granulomatous panuveitis is seen in?
Correct
- Bilateral granulomatous panuveitis is seen in Heerfordtâs syndrome (Uveoparotid fever).
- It is also associated with painful enlargement of parotid glands.
- Cranial nerve palsy, fever, malaise and skin rashes are also seen.
Incorrect
- Bilateral granulomatous panuveitis is seen in Heerfordtâs syndrome (Uveoparotid fever).
- It is also associated with painful enlargement of parotid glands.
- Cranial nerve palsy, fever, malaise and skin rashes are also seen.
-
Question 176 of 300
176. Question
Appendages of the eyes include all except?
Correct
Appendages of the eyes (adnexa of the eye / accessory visual structures) are:
- Eyelids
- Eyebrows
- Conjunctiva
- Lacrimal apparatus
Incorrect
Appendages of the eyes (adnexa of the eye / accessory visual structures) are:
- Eyelids
- Eyebrows
- Conjunctiva
- Lacrimal apparatus
-
Question 177 of 300
177. Question
Bitemporal hemianopia is a characteristic feature of?
Correct
Incorrect
-
Question 178 of 300
178. Question
Diplopia in superior oblique palsy is?
Correct
Incorrect
-
Question 179 of 300
179. Question
Which of the following is a topical NSAID for ophthalmic use?
Correct
Incorrect
-
Question 180 of 300
180. Question
An 18 year old boy comes to the eye casuality with history of injury with a tennis ball. On examination there is no perforation but there is hyphaema. The most likely source of the blood is
Correct
Incorrect
-
Question 181 of 300
181. Question
Treatment of alcohol dependence is by all except?
Correct
Incorrect
-
Question 182 of 300
182. Question
40 year old female came to the OPD with complaints of with depressed mood, insomnia, loss of appetite and lack of interest in surroundings for the past 1 year. These symptoms appeared after loss in a business deal 1 year ago. Which is correct regarding the management?
Correct
The history is characteristic of depressive disorder. Antidepressant drugs are to be given. But there are no ideal antidepressants. The treatment should be based on the side effect profile of the drugs.
Incorrect
The history is characteristic of depressive disorder. Antidepressant drugs are to be given. But there are no ideal antidepressants. The treatment should be based on the side effect profile of the drugs.
-
Question 183 of 300
183. Question
Drug used for treatment of nocturnal enuresis is?
Correct
- Enuresis alarm systems are the preferred treatment for children with nocturnal enuresis.
- These detect the moisture of urine when there is nocturnal enuresis.
- An alarm is triggered which wakes up the child.
- These systems are of 3 types â wearable alarms, wireless alarms and pad type alarms.
- Alarm systems are also have the least relapse rate among the various modalities of treatment.
- Motivational therapy
- The child is rewarded for keeping dry at night. But punishment for bedwetting is discouraged.
- There is limited evidence to support the efficacy of this approach.
- Drug therapy
- Pharmacological treatment for nocturnal enuresis is reserved for cases which do not respond to treatment with enuresis alarms.
- Desmopressin in the preferred drug. It is given at night to reduce urine production during sleep.
- Tricyclic antidepressants like imipramine can also be used. But use is limited due to side effects.
Incorrect
- Enuresis alarm systems are the preferred treatment for children with nocturnal enuresis.
- These detect the moisture of urine when there is nocturnal enuresis.
- An alarm is triggered which wakes up the child.
- These systems are of 3 types â wearable alarms, wireless alarms and pad type alarms.
- Alarm systems are also have the least relapse rate among the various modalities of treatment.
- Motivational therapy
- The child is rewarded for keeping dry at night. But punishment for bedwetting is discouraged.
- There is limited evidence to support the efficacy of this approach.
- Drug therapy
- Pharmacological treatment for nocturnal enuresis is reserved for cases which do not respond to treatment with enuresis alarms.
- Desmopressin in the preferred drug. It is given at night to reduce urine production during sleep.
- Tricyclic antidepressants like imipramine can also be used. But use is limited due to side effects.
-
Question 184 of 300
184. Question
Most common cause of premature death in schizophrenia is?
Correct
PREMATURE DEATH IN SCHIZOPHRENIA
- Life expectancy of patients with schizophrenia is decreased by approximately 15 to 25 years. 1
- Suicide is the most important cause of premature death in patients with schizophrenia.
- There is a 20 fold increased risk of suicide compared to the general population.
- Suicide attempts may occur without warning or expression of intent.
- Cancer is the second most frequent cause of mortality in schizophrenics. 2
- The rates of metabolic syndrome and cardiovascular disorders are also increased. 2
Incorrect
PREMATURE DEATH IN SCHIZOPHRENIA
- Life expectancy of patients with schizophrenia is decreased by approximately 15 to 25 years. 1
- Suicide is the most important cause of premature death in patients with schizophrenia.
- There is a 20 fold increased risk of suicide compared to the general population.
- Suicide attempts may occur without warning or expression of intent.
- Cancer is the second most frequent cause of mortality in schizophrenics. 2
- The rates of metabolic syndrome and cardiovascular disorders are also increased. 2
-
Question 185 of 300
185. Question
All are features of autistic disorders except ?
Correct
Incorrect
-
Question 186 of 300
186. Question
An 18 year old student complains of lack of interest in studies for last 6 months. He has frequent quarrels with his parents and has frequent headaches. The most appropriate clinical approach would be to:
Correct
The given features are not something that may be considered as normal adolescent problem. Also the headache described here cannot be classified under migraine. Oppositional defiant disorder often presents at a younger age. Hence it is necessary to rule out depression.
Incorrect
The given features are not something that may be considered as normal adolescent problem. Also the headache described here cannot be classified under migraine. Oppositional defiant disorder often presents at a younger age. Hence it is necessary to rule out depression.
-
Question 187 of 300
187. Question
One of the following usually differentiates hysterical symptoms from hypochondriacal symptoms:
Correct
Hysteria (Conversion disroder) is a condition in which a person has blindness, paralysis, or other nervous system (neurologic) symptoms that cannot be explained by medical evaluation.
Hypochondria is a belief that physical symptoms are signs of a serious illness, even when there is no medical evidence to support the presence of an illness
Incorrect
Hysteria (Conversion disroder) is a condition in which a person has blindness, paralysis, or other nervous system (neurologic) symptoms that cannot be explained by medical evaluation.
Hypochondria is a belief that physical symptoms are signs of a serious illness, even when there is no medical evidence to support the presence of an illness
-
Question 188 of 300
188. Question
A middle aged person reported to Psychiatric OPD with the complaints of the fear of leaving home, fear of travelling alone and fear of being in a crowd. He develops marked anxiety with palpitations and swelling if he is in these situations. He often avoids public transport to go to his place of work. The most likely diagnosis is:
Correct
Incorrect
-
Question 189 of 300
189. Question
A patient with acute psychosis, who is on haloperidol 20 mg/day for last 2 days, has an episode characterized by tongue protrusion, oculogyric crisis, stiffness and abnormal posture of limbs and trunk without loss of consciousness for last 20 minutes before presenting to casualty. This improved within a few minutes after
administration of diphenhydramine HCl. The most likely diagnosis is:Correct
Incorrect
-
Question 190 of 300
190. Question
The quantity of hormone released by Progestasert per day is?
Correct
Incorrect
-
Question 191 of 300
191. Question
Which of the following is true regarding Organ of Rosenmuller?
Correct
- Organ of Rosenmuller (epoophoron) represents cranial end of the Wolffian body.
- It consists of vertical tubules in the mesovarium and mesosalpinx.
- The tubule is lined by cubical cells.
- Paroophoron sometimes forms paraovarian cyst.
Incorrect
- Organ of Rosenmuller (epoophoron) represents cranial end of the Wolffian body.
- It consists of vertical tubules in the mesovarium and mesosalpinx.
- The tubule is lined by cubical cells.
- Paroophoron sometimes forms paraovarian cyst.
-
Question 192 of 300
192. Question
Tumour secreting placental alkaline phosphatase is?
Correct
- Dysgerminoma corresponds to seminoma of the testis.
- It is usually seen in young women or children.
- It secretes placental alkaline phosphatase.
- It does not secrete male of female sex hormones.
Ref: Shawâs Textbook of Gynaecology, 15th edition, p378.
Incorrect
- Dysgerminoma corresponds to seminoma of the testis.
- It is usually seen in young women or children.
- It secretes placental alkaline phosphatase.
- It does not secrete male of female sex hormones.
Ref: Shawâs Textbook of Gynaecology, 15th edition, p378.
-
Question 193 of 300
193. Question
All of the following are germ cell tumours except?
Correct
- Germ cell tumours include Teratoma, Dysgerminoma, Endodermal sinus tumour, Embryonal carcinoma, Polyembryoma, Choriocarcinoma and mixed forms.
- Mesonephroid tumour is an epithelial tumour.
- It is also known as clear cell carcinoma.
- It is an uncommon tumour of ovary.
- It is highly malignant.
Ref: Shawâs Textbook of Gynaecology, 15th edition, p373-376.
Incorrect
- Germ cell tumours include Teratoma, Dysgerminoma, Endodermal sinus tumour, Embryonal carcinoma, Polyembryoma, Choriocarcinoma and mixed forms.
- Mesonephroid tumour is an epithelial tumour.
- It is also known as clear cell carcinoma.
- It is an uncommon tumour of ovary.
- It is highly malignant.
Ref: Shawâs Textbook of Gynaecology, 15th edition, p373-376.
-
Question 194 of 300
194. Question
Which of the following is not true regarding Large Loop Excision of the Transformation Zone (LLETZ)?
Correct
- Large Loop Excision of the Transformation Zone (LLETZ) uses low voltage diathermy.
- It is used in the treatment of cervical intraepithelial neoplasias.
- It is used only for cervical lesions.
- Loop size lesser than 2cm gives a better cone.
- It is less costly and has shorter procedure time compared to laser ablation.
- But the success rate is similar to laser ablation.
Ref: Shawâs Textbook of Gynaecology, 15th edition, p406.
Incorrect
- Large Loop Excision of the Transformation Zone (LLETZ) uses low voltage diathermy.
- It is used in the treatment of cervical intraepithelial neoplasias.
- It is used only for cervical lesions.
- Loop size lesser than 2cm gives a better cone.
- It is less costly and has shorter procedure time compared to laser ablation.
- But the success rate is similar to laser ablation.
Ref: Shawâs Textbook of Gynaecology, 15th edition, p406.
-
Question 195 of 300
195. Question
Solid tumour with cystic areas in between filled with haemorrhagic fluid and accounting for about 20% of all ovarian cancers is?
Correct
- Endometrioid tumours account for about 20% of all ovarian cancers.
- They are mostly malignant.
- They are lined by glandular epithelium.
- They are moderate sized solid tumours with cystic areas in between filled with haemorrhagic fluid.
- Ovarian endometriosis coexist in 15% of cases.
Ref: Shawâs Textbook of Gynaecology, 15th edition, p375.
Incorrect
- Endometrioid tumours account for about 20% of all ovarian cancers.
- They are mostly malignant.
- They are lined by glandular epithelium.
- They are moderate sized solid tumours with cystic areas in between filled with haemorrhagic fluid.
- Ovarian endometriosis coexist in 15% of cases.
Ref: Shawâs Textbook of Gynaecology, 15th edition, p375.
-
Question 196 of 300
196. Question
Moschcowitz repair is done for?
Correct
- In Moschcowitz repair, several purse string sutures starting from below are used to obliterate the cul-de-sac of the pouch of Douglas.
- Care should be taken not to include the ureter in the stitch.
Ref: Shawâs Textbook of Gynaecology, 15th edition, p342.
Incorrect
- In Moschcowitz repair, several purse string sutures starting from below are used to obliterate the cul-de-sac of the pouch of Douglas.
- Care should be taken not to include the ureter in the stitch.
Ref: Shawâs Textbook of Gynaecology, 15th edition, p342.
-
Question 197 of 300
197. Question
Which is true regarding Savage syndrome?
Correct
- In Savage syndrome, there is receptor defect to gonadotrophic hormones.
- The clinical features resemble autoimmune disease.
- Height is normal.
- The ovaries contain follicles.
- FSH is raised.
Incorrect
- In Savage syndrome, there is receptor defect to gonadotrophic hormones.
- The clinical features resemble autoimmune disease.
- Height is normal.
- The ovaries contain follicles.
- FSH is raised.
-
Question 198 of 300
198. Question
All are true regarding medical treatment of endometriosis except?Correct
- Gestrinone has a long half life.
- It is a 19-nortestosterone derivative.
- It is given orally 2.5 to 5 mg twice weekly for 6 to 9 months.
- It has androgenic, antiprogestagenic, antioestrogenic and antigonadotrophic action.
Ref: Textbook of Gynaecology, Sheila Balakrishnan, 1st edition, p150.
Incorrect
- Gestrinone has a long half life.
- It is a 19-nortestosterone derivative.
- It is given orally 2.5 to 5 mg twice weekly for 6 to 9 months.
- It has androgenic, antiprogestagenic, antioestrogenic and antigonadotrophic action.
Ref: Textbook of Gynaecology, Sheila Balakrishnan, 1st edition, p150.
-
Question 199 of 300
199. Question
The use of which of the following drugs for the treatment of menorrhagia is not supported by clinical evidence?
Correct
Studies have failed to prove the effectiveness of ethamsylate in the treatment of menorrhagia.
Incorrect
Studies have failed to prove the effectiveness of ethamsylate in the treatment of menorrhagia.
-
Question 200 of 300
200. Question
Gold standard test for measuring à HCG levels in serum is?
Correct
Radioimmunoassay is the most sensitive test for measurement of à HCG levels in serum.
Incorrect
Radioimmunoassay is the most sensitive test for measurement of à HCG levels in serum.
-
Question 201 of 300
201. Question
False regarding hormone levels in polycystic ovarian disease?
Correct
HORMONE LEVELS IN POLYCYSTIC OVARIAN DISEASE
- Increased Androgen
- Increased Estrogen
- Increased Luteinizing hormone
- Decreased Follicular stimulating hormone
- Increased LH/FSH ratio (>2)
- Increased insulin
Incorrect
HORMONE LEVELS IN POLYCYSTIC OVARIAN DISEASE
- Increased Androgen
- Increased Estrogen
- Increased Luteinizing hormone
- Decreased Follicular stimulating hormone
- Increased LH/FSH ratio (>2)
- Increased insulin
-
Question 202 of 300
202. Question
Most specific marker for neural tube defects is?
Correct
Both alpha-fetoprotein and acetylcholinesterase in amniotic fluid are markers for neural tube defects. But acetylcholinesterase is the most specific marker.
Incorrect
Both alpha-fetoprotein and acetylcholinesterase in amniotic fluid are markers for neural tube defects. But acetylcholinesterase is the most specific marker.
-
Question 203 of 300
203. Question
A 13-year old girl came to the casualty with acute lower abdominal pain. She had cyclical pain for the last 6 months. She has not yet attained her menarche. On examination, a tense bulge was seen in the region of the hymen. The most probable diagnosis is?
Correct
- Imperforate hymen prevents the drainage of menstrual blood.
- This results in the accumulation of blood in the vagina leading to hematocolpos.
- This is visualised as the tense bulge.
- In long standing cases it can even lead to hematometra.
Incorrect
- Imperforate hymen prevents the drainage of menstrual blood.
- This results in the accumulation of blood in the vagina leading to hematocolpos.
- This is visualised as the tense bulge.
- In long standing cases it can even lead to hematometra.
-
Question 204 of 300
204. Question
A 27-year old primigravida presented to the OPD with pregnancy induced hypertension at 32 weeks of gestation. She had no other complications. On examination she had a blood pressure of 150/100 mm of Hg. Her blood pressure was later controlled with treatment. lf there are no complications, the pregnancy should be terminated at?
Correct
Incorrect
-
Question 205 of 300
205. Question
Weight gain in pregnancy depends on all except?
Correct
FACTORS INFLUENCING WEIGHT GAIN IN PREGNANCY
- Maternal age â Increased weight gain in younger women
- Physical activity â Several studies have demonstrated an inverse relation between weight gain and level of physical activity
- Prepregnancy weight â Weight gain in pregnancy is generally inversely proportional to BMI (Body Mass Index) in the pre pregnancy period
- Race / ethnicity â The mean weight gain differs in various ethnic / racial groups
- Parity â Lower weight gain was noted in multipara
- Hormonal milieu â Levels of hormones like insulin and leptin can influence weight gain in pregnancy
- Multiple pregnancy â Tend to gain more weight in multiple pregnancy
- Hyperemesis gravidarum â Lower weight gain in women with hyperemesis gravidarum
- Anorexia nervosa â Lower weight gain and lower birth weight in anorexic women
- Substance abuse â Studies have found no significant difference in mean weight gain between smoking and non smoking women. But the birth weight was lower in babies born to women who were smokers.
- Family violence â Greater risk of inadequate weight gain
- Marital status â Many studies have found that married women were more likely to gain weight than single / divorced women
Incorrect
FACTORS INFLUENCING WEIGHT GAIN IN PREGNANCY
- Maternal age â Increased weight gain in younger women
- Physical activity â Several studies have demonstrated an inverse relation between weight gain and level of physical activity
- Prepregnancy weight â Weight gain in pregnancy is generally inversely proportional to BMI (Body Mass Index) in the pre pregnancy period
- Race / ethnicity â The mean weight gain differs in various ethnic / racial groups
- Parity â Lower weight gain was noted in multipara
- Hormonal milieu â Levels of hormones like insulin and leptin can influence weight gain in pregnancy
- Multiple pregnancy â Tend to gain more weight in multiple pregnancy
- Hyperemesis gravidarum â Lower weight gain in women with hyperemesis gravidarum
- Anorexia nervosa â Lower weight gain and lower birth weight in anorexic women
- Substance abuse â Studies have found no significant difference in mean weight gain between smoking and non smoking women. But the birth weight was lower in babies born to women who were smokers.
- Family violence â Greater risk of inadequate weight gain
- Marital status â Many studies have found that married women were more likely to gain weight than single / divorced women
-
Question 206 of 300
206. Question
Which of the following congenital malformations can be diagnosed in the first trimester ?
Correct
CONGENITAL MALFORMATIONS DIAGNOSED IN THE FIRST TRIMESTER
- Routine ultrasound scan for the diagnosis of congenital anomalies is done in the second trimester
- But in high risk cases, an early anomaly scan is done to identify major anomalies
- The congenital anomalies that can be identified in the first trimester are:
- Acrania â Condition in which the flat bones of the cranial vault are partially or completely absent. It is an early stage of anencephaly.
- Exencephaly â An early stage of anencephaly in which the brain is located outside the skull
- Anencephaly â Absence of a major part of the brain, skull and scalp. It is an anomaly that occurs when the rostral end of the neural tube fails to close.
Incorrect
CONGENITAL MALFORMATIONS DIAGNOSED IN THE FIRST TRIMESTER
- Routine ultrasound scan for the diagnosis of congenital anomalies is done in the second trimester
- But in high risk cases, an early anomaly scan is done to identify major anomalies
- The congenital anomalies that can be identified in the first trimester are:
- Acrania â Condition in which the flat bones of the cranial vault are partially or completely absent. It is an early stage of anencephaly.
- Exencephaly â An early stage of anencephaly in which the brain is located outside the skull
- Anencephaly â Absence of a major part of the brain, skull and scalp. It is an anomaly that occurs when the rostral end of the neural tube fails to close.
-
Question 207 of 300
207. Question
Causes of primary amenorrhoea are all except
Correct
Sheehanâs syndrome results in secondary amenorrhoea.
CAUSES OF PRIMARY AMENORRHOEA
- Turner syndrome (45 XO)
- Swyer syndrome (46 XY, streak gonads, female phenotype)
- Testicular feminisation syndrome (46 XY, insensitivity to androgen, female phenotype)
- Mullerian agenesis ( Mayer Rokitansky Kuster Hauser [MRKH] syndrome)
- Cryptomenorrhea â Imperforate hymen / vaginal septum
- Kallmann syndrome (Hypogonadotropic hypogonadism with hyposmia / anosmia)
- Pituitary neoplasms
Incorrect
Sheehanâs syndrome results in secondary amenorrhoea.
CAUSES OF PRIMARY AMENORRHOEA
- Turner syndrome (45 XO)
- Swyer syndrome (46 XY, streak gonads, female phenotype)
- Testicular feminisation syndrome (46 XY, insensitivity to androgen, female phenotype)
- Mullerian agenesis ( Mayer Rokitansky Kuster Hauser [MRKH] syndrome)
- Cryptomenorrhea â Imperforate hymen / vaginal septum
- Kallmann syndrome (Hypogonadotropic hypogonadism with hyposmia / anosmia)
- Pituitary neoplasms
-
Question 208 of 300
208. Question
A primigravida in the first trimester of pregnancy was found to be sputum positive for acid fast bacilli. There is no prior history of tuberculosis. What is the treatment of choice for this patient?
Correct
ANTI TUBERCULOSIS TREATMENT IN PREGNANCY
- Newly smear positive cases of Tuberculosis are treated with Category I DOTS(Directly Observed Treatment Short course) in India.
- The drugs given in category I DOTS â Isoniazid, Rifampicin, Ethambutol and Pyrazinamide are not contraindicated in pregnancy
- But if the disease is not treated properly, it is harmful for the foetus
- Streptomycin is not given in pregnant patients as it is teratogenic
Incorrect
ANTI TUBERCULOSIS TREATMENT IN PREGNANCY
- Newly smear positive cases of Tuberculosis are treated with Category I DOTS(Directly Observed Treatment Short course) in India.
- The drugs given in category I DOTS â Isoniazid, Rifampicin, Ethambutol and Pyrazinamide are not contraindicated in pregnancy
- But if the disease is not treated properly, it is harmful for the foetus
- Streptomycin is not given in pregnant patients as it is teratogenic
-
Question 209 of 300
209. Question
Drug of choice for treatment of intrahepatic cholestasis in pregnancy is ?
Correct
Ursodeoxycholic acid decreases bile salt levels and relieves pruritus. It can also decrease the chance for fetal complications.
Incorrect
Ursodeoxycholic acid decreases bile salt levels and relieves pruritus. It can also decrease the chance for fetal complications.
-
Question 210 of 300
210. Question
Virus responsible for non immune hydrops foetalis is?
Correct
Parvovirus is the commonest infectious etiology for hydrops foetalis. The other viruses implicated include Cytomegalovirus, Hepatitis B virus, Herpes simplex virus, Rubella virus and Adenovirus.
Incorrect
Parvovirus is the commonest infectious etiology for hydrops foetalis. The other viruses implicated include Cytomegalovirus, Hepatitis B virus, Herpes simplex virus, Rubella virus and Adenovirus.
-
Question 211 of 300
211. Question
Increased nuchal translucency in 13th week fetal ultrasound characteristic of ?
Correct
Downâs syndrome is the most common cause of increased nuchal translucency in fetal ultrasound. The other causes are Trisomy 13, Trisomy 18, Klinefelter syndrome & Turnerâs syndrome.
Incorrect
Downâs syndrome is the most common cause of increased nuchal translucency in fetal ultrasound. The other causes are Trisomy 13, Trisomy 18, Klinefelter syndrome & Turnerâs syndrome.
-
Question 212 of 300
212. Question
Which among the following is most commonly associated with carcinoma cervix?
Correct
HPV 16 is most commonly associated with carcinoma cervix. It is responsible for 40-70% of all invasive squamous cell cervical cancers
Incorrect
HPV 16 is most commonly associated with carcinoma cervix. It is responsible for 40-70% of all invasive squamous cell cervical cancers
-
Question 213 of 300
213. Question
Patients of Rectovaginal fistula should be initially treated with:
Correct
The definitive treatment is surgery. But to increase the success rate of surgery, colostomy can be done initially. This decreases inflammation.
Incorrect
The definitive treatment is surgery. But to increase the success rate of surgery, colostomy can be done initially. This decreases inflammation.
-
Question 214 of 300
214. Question
The most common cause of secondary amenorrhoea in India is:
Correct
Incorrect
-
Question 215 of 300
215. Question
The following drug is not helpful in the treatment of ectopic pregnancy:
Correct
Misoprostol can be used for early abortion. But it is not useful in ectopic pregnancy.
Incorrect
Misoprostol can be used for early abortion. But it is not useful in ectopic pregnancy.
-
Question 216 of 300
216. Question
Minimum depth for the lining of a sanitary well is?
Correct
- The lining of a sanitary well should go down at least 20 feet to ensure that water enters from the bottom and not from the sides of the well.
- The lining should extend upwards for about 2-3 feet above the ground.
Ref: Parkâs Textbook of Preventive and Social Medicine, 20th edition, p620
Incorrect
- The lining of a sanitary well should go down at least 20 feet to ensure that water enters from the bottom and not from the sides of the well.
- The lining should extend upwards for about 2-3 feet above the ground.
Ref: Parkâs Textbook of Preventive and Social Medicine, 20th edition, p620
-
Question 217 of 300
217. Question
Quantity of water available per capita considered adequate to meet the needs of all urban domestic purposes?
Correct
- 2 L is the amount of water required for meeting the physiological requirements of the body everyday.
- But water is also required for many other activities of daily life such as cooking, washing, bathing etc.
- Taking this into consideration, 150-200 L of water per capita is the amount considered adequate to meet the needs of all urban domestic purposes.
Ref: Parkâs Textbook of Preventive and Social Medicine, 20th edition, p617
Incorrect
- 2 L is the amount of water required for meeting the physiological requirements of the body everyday.
- But water is also required for many other activities of daily life such as cooking, washing, bathing etc.
- Taking this into consideration, 150-200 L of water per capita is the amount considered adequate to meet the needs of all urban domestic purposes.
Ref: Parkâs Textbook of Preventive and Social Medicine, 20th edition, p617
-
Question 218 of 300
218. Question
Screening test specificity is the ability of a test to correctly identify:
Correct
- Specificity of a test is the ability to correctly identify True negatives. It is calculated as follows:
- Specificity = Number of true negatives / (Number of true negatives + Number of false positives )
Incorrect
- Specificity of a test is the ability to correctly identify True negatives. It is calculated as follows:
- Specificity = Number of true negatives / (Number of true negatives + Number of false positives )
-
Question 219 of 300
219. Question
All are true about randomized control trial except?
Correct
Incorrect
-
Question 220 of 300
220. Question
A sewage worker with fever and jaundice presented to the emergency department. Lab investigations revealed increased blood urea nitrogen and serum creatinine â indicative of renal failure. Which of the following antibiotics is preferred in this patient?
Correct
- The history of fever and jaundice in a sewage worker with features of renal failure is suggestive of Weilâs disease
- Intravenous penicillin G is the preferred antibiotic for treatment of Weilâs disease
- Erythromycin can be used as an alternative
- In mild cases of leptospirosis, oral therapy with ampicillin / tetracycline is given
- Doxycycline is used for prophylaxis against leptospirosis
Incorrect
- The history of fever and jaundice in a sewage worker with features of renal failure is suggestive of Weilâs disease
- Intravenous penicillin G is the preferred antibiotic for treatment of Weilâs disease
- Erythromycin can be used as an alternative
- In mild cases of leptospirosis, oral therapy with ampicillin / tetracycline is given
- Doxycycline is used for prophylaxis against leptospirosis
-
Question 221 of 300
221. Question
Modes of disease transmission by vectors are all except?
Correct
Modes of disease transmission by vectors are:
- Biting
- Regurgitation of ingested material
- Rubbing of of infected excrement
- Contamination by body fluids of the vector
Incorrect
Modes of disease transmission by vectors are:
- Biting
- Regurgitation of ingested material
- Rubbing of of infected excrement
- Contamination by body fluids of the vector
-
Question 222 of 300
222. Question
Tetracycline is used for the prophylaxis of?
Correct
Drug of choice for chemoprophylaxis of cholera is tetracycline. Doxycycline can also be used. Chemoprophylaxis is given for household contacts of cholera patients / in case of an outbreak in a closed community. But it is not given in case of mass outbreaks of cholera.
Incorrect
Drug of choice for chemoprophylaxis of cholera is tetracycline. Doxycycline can also be used. Chemoprophylaxis is given for household contacts of cholera patients / in case of an outbreak in a closed community. But it is not given in case of mass outbreaks of cholera.
-
Question 223 of 300
223. Question
Mass chemoprophylaxis in an endemic area is given for all except?
Correct
Incorrect
-
Question 224 of 300
224. Question
Mass chemoprophylaxis in an endemic area is given for all except?
Correct
Incorrect
-
Question 225 of 300
225. Question
A case of Acute Flaccid Paralysis should be kept under surveillance for residual paralysis for?
Correct
Incorrect
-
Question 226 of 300
226. Question
The usefulness of a âscreening test` in a community depends on its :
Correct
A screening test should be sensitive to detect maximum possible cases. Specificity is not as important. After screening, we can apply a confirmatory test with high specificity.
Incorrect
A screening test should be sensitive to detect maximum possible cases. Specificity is not as important. After screening, we can apply a confirmatory test with high specificity.
-
Question 227 of 300
227. Question
âFive clean practicesâ under strategies for elimination of neonatal tetanus include all except :
Correct
The 5 clean practices are:
- Clean hands
- Clean delivery surface
- Clean cord care
- Clean blade for cutting cord
- Clean cord tie and no application on cord stump
Incorrect
The 5 clean practices are:
- Clean hands
- Clean delivery surface
- Clean cord care
- Clean blade for cutting cord
- Clean cord tie and no application on cord stump
-
Question 228 of 300
228. Question
The following statements are true about Intrauterine devices (IUD) except:
Correct
Multiload Cu-375 is a second generation IUD. Progestasert and Levonova are third generation IUCDâs.
Incorrect
Multiload Cu-375 is a second generation IUD. Progestasert and Levonova are third generation IUCDâs.
-
Question 229 of 300
229. Question
The following statements about breast milk are true except :
Correct
Incorrect
-
Question 230 of 300
230. Question
The current recommendation for breast feeding is that :
Correct
Incorrect
-
Question 231 of 300
231. Question
All of the following statements about leprosy are true except :
Correct
Incorrect
-
Question 232 of 300
232. Question
A 37 weeks pregnant woman attends an antenatal clinic at a Primary Health Centre. She has not had any antenatal visit till now. The best approach regarding tetanus immunization in this case would be to
Correct
Tetanus toxoid should be given to a pregnant women irrespective of the month of pregnancy. Second dose must be given atleast 4 weeks later. The newborn will not be protected unless 2 doses are taken. Newborns born to unimmunized mothers can be protected by giving tetanus antitoxin (750IU).
Incorrect
Tetanus toxoid should be given to a pregnant women irrespective of the month of pregnancy. Second dose must be given atleast 4 weeks later. The newborn will not be protected unless 2 doses are taken. Newborns born to unimmunized mothers can be protected by giving tetanus antitoxin (750IU).
-
Question 233 of 300
233. Question
Rose-Waller test is a?
Correct
Incorrect
-
Question 234 of 300
234. Question
The vaccine for Japanese Encephalitis is?
Correct
- The vaccine for Japanese Encephalitis is a killed vaccine.
- The virus is grown in mouse brain and is inactivated by formalin.
- Influenza vaccine can be of various types â Live (mutant), Live (attenuated) and Live (recombinant) forms.
Ref: Ananthanarayan and Panikerâs Textbook of Microbiology, 9th edition, p452.
Incorrect
- The vaccine for Japanese Encephalitis is a killed vaccine.
- The virus is grown in mouse brain and is inactivated by formalin.
- Influenza vaccine can be of various types â Live (mutant), Live (attenuated) and Live (recombinant) forms.
Ref: Ananthanarayan and Panikerâs Textbook of Microbiology, 9th edition, p452.
-
Question 235 of 300
235. Question
Which of the following is a human RNA virus?
Correct
Incorrect
-
Question 236 of 300
236. Question
All are true regarding superantigens except?
Correct
- Superantigens activate very large number of T cells.
- They are medium sized proteins which bind outside the antibody binding groove.
- They have high resistance to proteases.
- They cause release of cytokines resulting in massive proliferation of T lymphocytes.
Ref: Ananthanarayan and Panikerâs Textbook of Microbiology, 9th edition, p90.
Incorrect
- Superantigens activate very large number of T cells.
- They are medium sized proteins which bind outside the antibody binding groove.
- They have high resistance to proteases.
- They cause release of cytokines resulting in massive proliferation of T lymphocytes.
Ref: Ananthanarayan and Panikerâs Textbook of Microbiology, 9th edition, p90.
-
Question 237 of 300
237. Question
Which of the following is an intermediate level disinfectant?
Correct
Incorrect
-
Question 238 of 300
238. Question
The best culture medium for primary isolation of H. influenzae is?
Correct
- Fildesâ agar is made by adding peptic digest of blood to nutrient agar.
- It gives copious growth of H. influenzae and it is the best culture medium for the primary isolation of H. influenzae.
- The other media used for the isolation of H. influenzae are chocolate agar, blood agar, nutrient agar and Levinthalâs medium.
Ref: Ananthanarayan and Panikerâs Textbook of Microbiology, 9th edition, p330.
Incorrect
- Fildesâ agar is made by adding peptic digest of blood to nutrient agar.
- It gives copious growth of H. influenzae and it is the best culture medium for the primary isolation of H. influenzae.
- The other media used for the isolation of H. influenzae are chocolate agar, blood agar, nutrient agar and Levinthalâs medium.
Ref: Ananthanarayan and Panikerâs Textbook of Microbiology, 9th edition, p330.
-
Question 239 of 300
239. Question
Which of the following is true regarding anthracoid bacilli?
Correct
- Anthracoid bacilli usually grow in penicillin agar.
- Their growth is not inhibited by chloral hydrate.
- They usually grow at 45 degree celsius.
- They are not susceptible to gamma phage.
- They are motile, non capsulated bacilli which grow in short chains.
Incorrect
- Anthracoid bacilli usually grow in penicillin agar.
- Their growth is not inhibited by chloral hydrate.
- They usually grow at 45 degree celsius.
- They are not susceptible to gamma phage.
- They are motile, non capsulated bacilli which grow in short chains.
-
Question 240 of 300
240. Question
Which of the following gastroenteritis viruses can spread by air?
Correct
Incorrect
-
Question 241 of 300
241. Question
Most common fungal infection in immunocompetent individuals is?
Correct
- Candidiasis is the most common fungal infection in immunocompetent individuals.
- All of the infections mentioned above are usually seen in immunocompromised patients, but they are also seen in immunocompetent persons.
- Candida is part of the normal flora of skin and mucous membranes.
- When the normal flora is altered by the use of antibiotics  / other factors, candida can cause infection.
Incorrect
- Candidiasis is the most common fungal infection in immunocompetent individuals.
- All of the infections mentioned above are usually seen in immunocompromised patients, but they are also seen in immunocompetent persons.
- Candida is part of the normal flora of skin and mucous membranes.
- When the normal flora is altered by the use of antibiotics  / other factors, candida can cause infection.
-
Question 242 of 300
242. Question
Erythema chronicum migrans is caused by?
Correct
- Erythema chronicum migrans is a rash often seen in the early stages of Lyme disease.
- It is caused by the spirochete Borrelia burgdorferi.
- Lyme disease is a zoonosis which is spread to humans by tick bite
- The rash appears a few days after the tick bite
Incorrect
- Erythema chronicum migrans is a rash often seen in the early stages of Lyme disease.
- It is caused by the spirochete Borrelia burgdorferi.
- Lyme disease is a zoonosis which is spread to humans by tick bite
- The rash appears a few days after the tick bite
-
Question 243 of 300
243. Question
Draughtsman appearance is seen with colonies of:
Correct
Incorrect
-
Question 244 of 300
244. Question
False statement about streptococcus:
Correct
Incorrect
-
Question 245 of 300
245. Question
All of the following statements regarding Corynebacterium diphtheriae are true, except?
Correct
Incorrect
-
Question 246 of 300
246. Question
All of the following statements are true regarding poliovirus except :
Correct
Incorrect
-
Question 247 of 300
247. Question
False regarding Pneumococcus is?
Correct
Incorrect
-
Question 248 of 300
248. Question
False regarding Pneumococcus is?
Correct
Incorrect
-
Question 249 of 300
249. Question
False regarding Pneumococcus is?
Correct
Incorrect
-
Question 250 of 300
250. Question
A patient presented to the OPD with clinical features of pneumonia. Sputum examination of the patient revealed a gram positive cocci with alpha hemolysis on sheep agar. Which test will you do to confirm the diagnosis?
Correct
- The clinical features and microbiological characteristics are suggestive of pneumococcal pneumonia
- The diagnosis can be confirmed by Bile solubility and Optochin sensitivity
Incorrect
- The clinical features and microbiological characteristics are suggestive of pneumococcal pneumonia
- The diagnosis can be confirmed by Bile solubility and Optochin sensitivity
-
Question 251 of 300
251. Question
Epstein Barr (EB) virus has been implicated in the following malignancies except:
Correct
Epstein Barr (EB) virus associated conditions:
- Infectious mononucleosis
- Acute transverse myelitis
- Guillain Barre syndrome
- Aplastic anemia
- Chronic fatigue syndrome
- Peripheral neuritis
- T cell lymphoma
- Burkittâs lymphoma
- Hodgkinâs disease
- CNS lymphoma
- Lymphoproliferative syndrome
- Gastric carcinoma
- Leiomyosarcoma
- Tonsillar carcinoma
- Nasopharyngeal carcinoma
- Thymoma
- Oral hairy leukoplakia (in adults with HIV)
Incorrect
Epstein Barr (EB) virus associated conditions:
- Infectious mononucleosis
- Acute transverse myelitis
- Guillain Barre syndrome
- Aplastic anemia
- Chronic fatigue syndrome
- Peripheral neuritis
- T cell lymphoma
- Burkittâs lymphoma
- Hodgkinâs disease
- CNS lymphoma
- Lymphoproliferative syndrome
- Gastric carcinoma
- Leiomyosarcoma
- Tonsillar carcinoma
- Nasopharyngeal carcinoma
- Thymoma
- Oral hairy leukoplakia (in adults with HIV)
-
Question 252 of 300
252. Question
The most sensitive method for detecting cervical Chlamydia trachomatis infection is:
Correct
Incorrect
-
Question 253 of 300
253. Question
Best investigation for the detection of bone metastases is?
Correct
Incorrect
-
Question 254 of 300
254. Question
Which of the following contrast agents can be used in a patient with renal dysfunction for the prevention of contrast nephropathy?
Correct
PREVENTION OF CONTRAST NEPHROPATHY
- Non ionic low osmolar contrast agents are preferred in patients with decreased renal function to prevent contrast nephropathy
- Ionic contrast media have a higher risk for contrast nephropathy, and hence should be avoided
- The amount of contrast media used should be limited
- The patient should be well hydrated before the procedure
- Supplementary interventions include use of N-acetylcysteine, fenoldopam, theophylline, prostaglandins and magnesium
Incorrect
PREVENTION OF CONTRAST NEPHROPATHY
- Non ionic low osmolar contrast agents are preferred in patients with decreased renal function to prevent contrast nephropathy
- Ionic contrast media have a higher risk for contrast nephropathy, and hence should be avoided
- The amount of contrast media used should be limited
- The patient should be well hydrated before the procedure
- Supplementary interventions include use of N-acetylcysteine, fenoldopam, theophylline, prostaglandins and magnesium
-
Question 255 of 300
255. Question
Egg on side appearance is characteristic of?
Correct
Incorrect
-
Question 256 of 300
256. Question
Most ionizing radiation among the following is?
Correct
Incorrect
-
Question 257 of 300
257. Question
Which of the following substances are used to coat the walls of a CT scan room for radiation shielding?
Correct
Incorrect
-
Question 258 of 300
258. Question
Which one of the following tumors shows calcification on CT Scan:
Correct
Both meningioma and ependymoma show calcification. But it is more common in meningioma.
Incorrect
Both meningioma and ependymoma show calcification. But it is more common in meningioma.
-
Question 259 of 300
259. Question
Which one of the following is the most preferred route to perform cerebral angiography?
Correct
Incorrect
-
Question 260 of 300
260. Question
Which of the following ultrasound marker is associated with greatest increased risk for Trisomy 21 in fetus:
Correct
Incorrect
-
Question 261 of 300
261. Question
The most sensitive imaging modality for diagnosis of ureteric stones in a patient with acute colic is:
Correct
Incorrect
-
Question 262 of 300
262. Question
The gold standard for the diagnosis of osteoporosis is:
Correct
Incorrect
-
Question 263 of 300
263. Question
The technique employed in radiotherapy to counteract the effect of tumour motion due to breathing is known as
Correct
Respiratory gating is a technique in radiotherapy in which the radiation is applied during that phase of respiratory cycle in which the tumour is in the best range. This is to prevent unnecessary radiation exposure to normal structures.
Incorrect
Respiratory gating is a technique in radiotherapy in which the radiation is applied during that phase of respiratory cycle in which the tumour is in the best range. This is to prevent unnecessary radiation exposure to normal structures.
-
Question 264 of 300
264. Question
The MR imaging in multiple sclerosis will show lesions in:
Correct
Incorrect
-
Question 265 of 300
265. Question
The most common cause of acquired arteriovenous fistula is;
Correct
Incorrect
-
Question 266 of 300
266. Question
Which of the following statements is true regarding testicular tumors?
Correct
Incorrect
-
Question 267 of 300
267. Question
One of the following is not correct in papillary carcinoma of thyroid:
Correct
Incorrect
-
Question 268 of 300
268. Question
All of the following methods are antilarval measures except:
Correct
Incorrect
-
Question 269 of 300
269. Question
Which of the following viral infections is transmitted by tick?
Correct
- KFD is transmitted by bite of hard ticks
- Japanese encephalitis â Culex mosquito
- Dengue fever â Aedes mosquito
- Yellow fever â Aedes mosquito
Incorrect
- KFD is transmitted by bite of hard ticks
- Japanese encephalitis â Culex mosquito
- Dengue fever â Aedes mosquito
- Yellow fever â Aedes mosquito
-
Question 270 of 300
270. Question
Which one of the following statements about influence of smoking on risk of coronary heart disease (CHD) is not true?
Correct
Incorrect
-
Question 271 of 300
271. Question
Antibiotic treatment of choice for treating cholera in an adult is a single dose of:
Correct
Incorrect
-
Question 272 of 300
272. Question
For the treatment of case a of class III dog bite, all of the following are correct except:
Correct
Incorrect
-
Question 273 of 300
273. Question
Infant mortality does not include:
Correct
Infant mortality includes death within the first year of life. Perinatal mortality includes late foetal deaths (28 weeks of gestation to birth) and early neonatal deaths (birth to 7 days).
Other terms:
Late neonatal mortality â 7 days to 28 days
Neonatal mortality â birth to 28 days (early neonatal mortality + late neonatal mortality)
Post neonatal mortality â 28 days to 1 yearIncorrect
Infant mortality includes death within the first year of life. Perinatal mortality includes late foetal deaths (28 weeks of gestation to birth) and early neonatal deaths (birth to 7 days).
Other terms:
Late neonatal mortality â 7 days to 28 days
Neonatal mortality â birth to 28 days (early neonatal mortality + late neonatal mortality)
Post neonatal mortality â 28 days to 1 year -
Question 274 of 300
274. Question
All of the following indicators are included in Physical Quality of Life Index (PQLI) except:
Correct
Incorrect
-
Question 275 of 300
275. Question
True about ASHA (accredited social health activist) is
Correct
Incorrect
-
Question 276 of 300
276. Question
Lichtenberg figures are seen in?
Correct
Incorrect
-
Question 277 of 300
277. Question
Danbury tremor is seen in?
Correct
- Danbury tremor is a classical and consistent manifestation of mercury poisoning.
- It is a coarse intentional tremor interspersed with jerky movements.
- It begins in the hand and progresses to lips, tongue, arms and legs.
Ref: Textbook of Forensic Medicine and Toxicology, V. V. Pillay, 14th edition, p406.
Incorrect
- Danbury tremor is a classical and consistent manifestation of mercury poisoning.
- It is a coarse intentional tremor interspersed with jerky movements.
- It begins in the hand and progresses to lips, tongue, arms and legs.
Ref: Textbook of Forensic Medicine and Toxicology, V. V. Pillay, 14th edition, p406.
-
Question 278 of 300
278. Question
âParadox gunâ is a?
Correct
Incorrect
-
Question 279 of 300
279. Question
The combination of alopecia & skin rash, painful peripheral neuropathy and confusion with lethargy are seen in?
Correct
- The combination of alopecia & skin rash, painful peripheral neuropathy and confusion with lethargy are known as âThallium triadâ.
- It is indicative of chronic thallium poisoning.
- Other symptoms include dystrophy of nails (Meesâ lines; also seen in arsenic poisoning), cardiovascular symptoms like hypertension and cardiomyopathy, ophthalmological symptoms like ptosis, ophthalmoplegia, nystagmus, keratitis, lens opacity and optic atrophy.
Ref: Textbook of Forensic Medicine and Toxicology, V. V. Pillay, 14th edition, p414.
Incorrect
- The combination of alopecia & skin rash, painful peripheral neuropathy and confusion with lethargy are known as âThallium triadâ.
- It is indicative of chronic thallium poisoning.
- Other symptoms include dystrophy of nails (Meesâ lines; also seen in arsenic poisoning), cardiovascular symptoms like hypertension and cardiomyopathy, ophthalmological symptoms like ptosis, ophthalmoplegia, nystagmus, keratitis, lens opacity and optic atrophy.
Ref: Textbook of Forensic Medicine and Toxicology, V. V. Pillay, 14th edition, p414.
-
Question 280 of 300
280. Question
The test to determine blood groups in a blood stain is?
Correct
- The common blood groups ABO, MNS and Rh systems can be determined in a blood stain by Lattes crust method.
- Luminol spray is used to identify blood stains which are no longer visible due to washing of crime scene.
- Kastle-Meyer test is used to identify visible blood stains.
- Haemin crystal test is a confirmatory test for blood stains.
Ref: Textbook of Forensic Medicine and Toxicology, V. V. Pillay, 14th edition, p320-323.
Incorrect
- The common blood groups ABO, MNS and Rh systems can be determined in a blood stain by Lattes crust method.
- Luminol spray is used to identify blood stains which are no longer visible due to washing of crime scene.
- Kastle-Meyer test is used to identify visible blood stains.
- Haemin crystal test is a confirmatory test for blood stains.
Ref: Textbook of Forensic Medicine and Toxicology, V. V. Pillay, 14th edition, p320-323.
-
Question 281 of 300
281. Question
Rave drug is?
Correct
MDMA (3,4-methylenedioxy-N-methylamphetamine), widely known as âEcstasyâ is a psychoactive drug with stimulant and hallucinogenic effects. It produces euphoria. It is consumed by individuals attending clubs and parties.
Incorrect
MDMA (3,4-methylenedioxy-N-methylamphetamine), widely known as âEcstasyâ is a psychoactive drug with stimulant and hallucinogenic effects. It produces euphoria. It is consumed by individuals attending clubs and parties.
-
Question 282 of 300
282. Question
Which of the following is used for narcoanalysis?
Correct
Drugs used for narcoanalysis are:
- Scopolamine (hyoscine)
- Sodium thiopental (Sodium Pentothal)
- Amobarbital  (Amytal Sodium)
- Secobarbital sodium (Seconal)
Incorrect
Drugs used for narcoanalysis are:
- Scopolamine (hyoscine)
- Sodium thiopental (Sodium Pentothal)
- Amobarbital  (Amytal Sodium)
- Secobarbital sodium (Seconal)
-
Question 283 of 300
283. Question
A 34-year old rickshaw puller has been using heroin for the past ten years. One evening his family members found him unconscious. He was brought to the casualty. On examination he had tachycardia, shallow breathing, constricted pupils. His blood pressure was 100/70 mm of Hg. He had brisk bilateral deep tendon reflexes. The planter reflexes were flexor on both sides. Which of the following is the best treatment for him?
Correct
The clinical features are suggestive of acute heroin intoxication. The drug of choice is naloxone â an opioid antagonist. It is reduces the action of heroin. Buprenorphine is a partial opioid agonist. Flumazenil is a benzodiazepine antagonist. Methadone is used in rehabilitation of opoid addicts.
Incorrect
The clinical features are suggestive of acute heroin intoxication. The drug of choice is naloxone â an opioid antagonist. It is reduces the action of heroin. Buprenorphine is a partial opioid agonist. Flumazenil is a benzodiazepine antagonist. Methadone is used in rehabilitation of opoid addicts.
-
Question 284 of 300
284. Question
A 39 year old Carpenter has taken two bottles of liquor from the local shop. After about and hour, he develops confusion, vomiting and blurring of vision. He has been brought to the emergency department. He should be given
Correct
The clinical features are suggestive of methyl alcohol poisoning. Formic acid and formaldehyde (metabolites of methyl alcohol) are responsible for these symptoms. Ethyl alcohol is used for treatment as it saturates alcohol dehydrogenase, thereby decreasing production of formic acid and formaldehyde.
Incorrect
The clinical features are suggestive of methyl alcohol poisoning. Formic acid and formaldehyde (metabolites of methyl alcohol) are responsible for these symptoms. Ethyl alcohol is used for treatment as it saturates alcohol dehydrogenase, thereby decreasing production of formic acid and formaldehyde.
-
Question 285 of 300
285. Question
The most reliable criteria in Gustafsonâs method of identification is:
Correct
Incorrect
-
Question 286 of 300
286. Question
Which of the following is true regarding nuclear fluorescence in systemic lupus erythematosus?
Correct
Incorrect
-
Question 287 of 300
287. Question
Which of the following is true regarding lymphangioleiomyomatosis?
Correct
Incorrect
-
Question 288 of 300
288. Question
All are true about Xanthogranulomatous inflammation except?
Correct
- Xanthogranulomatous inflammation is a condition in which lipid laden macrophages (foam cells) are deposited in various parts of the body
- The exact etiopathogenesis is not known
- E.coli and proteus are implicated as etiological agents
- Mycobacterium tuberculosis is not considered as an etiological agent
Incorrect
- Xanthogranulomatous inflammation is a condition in which lipid laden macrophages (foam cells) are deposited in various parts of the body
- The exact etiopathogenesis is not known
- E.coli and proteus are implicated as etiological agents
- Mycobacterium tuberculosis is not considered as an etiological agent
-
Question 289 of 300
289. Question
Which of the following gene defect is associated with development of medullary carcinoma of thyroid?
Correct
Incorrect
-
Question 290 of 300
290. Question
Which of the following is a procoagulation protein?
Correct
Incorrect
-
Question 291 of 300
291. Question
Which one of the following is not a feature of liver histology in Non cirrhotic portal fibrosis (NCPF)?
Correct
Incorrect
-
Question 292 of 300
292. Question
The chances of having an unaffected baby, when both parents have achondroplasia, are:
Correct
Incorrect
-
Question 293 of 300
293. Question
Characteristic feature in kidney biopsy from a child with hemolytic uremic syndrome is:
Correct
Incorrect
-
Question 294 of 300
294. Question
In which of the following conditions bilateral contracted kidneys are characteristically seen?
Correct
Incorrect
-
Question 295 of 300
295. Question
Atypical pneumonia can be caused by the following microbial agents except?
Correct
Incorrect
-
Question 296 of 300
296. Question
Fibrinoid necrosis may be observed in all of the following, except:
Correct
Incorrect
-
Question 297 of 300
297. Question
All of the following statements are true regarding reversible cell injury, except:
Correct
Incorrect
-
Question 298 of 300
298. Question
The percentage of pulmonary emboli, that proceed to infarction is approximately:
Correct
Incorrect
-
Question 299 of 300
299. Question
All of the following features are seen in the viral pneumonia except:
Correct
Incorrect
-
Question 300 of 300
300. Question
A 17 year-old boy presented with TLC of 138 x 10 (9) /L with 80% blasts on the peripheral smear. Chest X-ray demonstrated a large mediastinal mass. Immunophenotyping of this patientâs blasts would most likely demonstrate:
Correct
There is increased lymphocytes with 80% blasts â indicating acute leukemia. Also the age group and the mediastinal mass point out to the possibility of T cell leukemia. (T cell leukemia is usually seen in adolescent boys with thymic involvement). Hence the blasts would demonstrate immature T cell phenotype
Incorrect
There is increased lymphocytes with 80% blasts â indicating acute leukemia. Also the age group and the mediastinal mass point out to the possibility of T cell leukemia. (T cell leukemia is usually seen in adolescent boys with thymic involvement). Hence the blasts would demonstrate immature T cell phenotype
To register click hereÂ